Download as pdf or txt
Download as pdf or txt
You are on page 1of 82

lOMoARcPSD|32626796

Management Advisory Services- Integrated Review Quizzes

BS Accountancy (New Era University)

Scan to open on Studocu

Studocu is not sponsored or endorsed by any college or university


Downloaded by Sky Park (parksky512@gmail.com)
lOMoARcPSD|32626796

1. When an organization is operating above the breakeven point, the degree of amount that sales may decline
before losses are incurred is called the
a. Residual Income Rate
b. Margin of Safety
c. Marginal Rate of Return
d. Target (Hurdle) Rate of Return
2. The responsibility for safeguarding financial assets and arranging financing js given to the
a. Controller
b. Comptroller
c. Chief Financial Officer
d. Treasurer
3. All of the following are examples of imputed costs except
a. The stated interest paid on a bank loan
b. The use of the firm's internal cash funds to purchase assets.
c. Assets that are considered obsolete that maintain a net book value.
d. Decelerated depreciation
4. Pinoy Company temporarily has excess production capacity, the idle plant facilities can be used to manufacture
a low- margin item. The low-margin item should be produced if it can be sold for more than its
a. Variable costs plus opportunity cost of idle facilities.
b. Indirect costs plus any opportunity cost of idle facilities.
c. fixed costs
d. Variable costs
5. In equipment-replacement decisions, which one of the following does not affect the decision-making process?
a. Current disposal price of the old equipment
b. Operating costs of the old equipment
c. Original fair market value of the old equipment.
d. Cost of the new equipment

Metropolis Manufacturing Corp, which produces two products for which the following data have been tabulated
Fixed manufacturing costs is applied at a rate of P 1.00 per machine hour
Per unit XY-7 BD-4
Selling price P 4.00 P 3.00
Variable manufacturing cost P 2.00 P 1.50
Fixed manufacturing cost P 0.75 P 0.20
Variable selling cost P 1.00 P 1.00
The sales manager has had a P 160,000 increase in the budget allotment for advertising and wants to apply the
money to the most profitable product. The products are not substitutes for one another in the eyes of the company's
customers

6. Suppose the sales manager chooses to devote the entire P 160,000 to increased advertising for XY-7. The
minimum increase in sales unit of XY-Z required to offset the increased advertising would be
a. 640,000 units
b. 160,000 units
c. 80,000 units
d. 128,000 units

Downloaded by Sky Park (parksky512@gmail.com)


lOMoARcPSD|32626796

The increase in unit sales to offset the increase in advertising.


XY-7 Increase in unit sales= Increase in fixed cost / CM
VC/unit = P3.00 (P2 + P1) = P160,000 / P1
CM= P1.00 (P4 3 P3). = 160,000 units

7. Suppose the sales manager chooses to devote the entire P 160,000 to increased advertising for BD-4. The
minimum increase in sales unit of BD-4 required to offset the increased advertising would be
a. P 160,00
b. P 320,000
c. P 960,000
d. P 1,600, 000
BD-4
VC/unit= P2.50 (P1.50 + P1.00) Increase in peso sales= Increase in fixed cost / CMR
CM= P0.50 (P3.00 3 P2.50). = P160,000 ÷ 1⁄6
= 1⁄6
0.50 �㔶�㕀
CM Rate= = P960,000
3 Ā�㕒�㕙�㕙�㕖�㕛�㕔 �㕝ÿ�㕖�㕐�㕒

8. Suppose Metropolis has only 100,000 machine hours that can be made available to produce XY-Z and BD-4.
If the potential increase in sales units for either product resulting from advertising is far in excess of these
production capabilities, which product should be advertised and what is the estimated increase in
contribution margin earned?
a. Product XY-Z should be produced, yielding a margin of P 75,000
b. Product XY-Z should be produced, yielding a contribution margin of P 133,333.
c. Product BD-4 should be produced. yielding a contribution margin of P 250,000,
d. Produt.t BD-4 should be produced, yielding contribution margin P 187,500
The unit contribution margin of each product must be converted into contribution margin per hour to
determine which is more profitable.
XY-Z BD-4
It is given that the fixed overhead is
UCM 1.00 0.50
applied at P1.00 per hour. The
÷ # of hours per unit
contribution margin per hour shall
(0.75 per unit/ 1 per hour) 0.75 hr
be computed as follows:
(0.20 per unit/ 1 per hour) 0.20 hr
CM per hour 1.33 2.50
Product BD-4 gives a higher CM per hour and should be prioritized for production and sales. The
total CM in producing BD-4 is P250,000 (i.e., 100,000 hours x P2.50 per hour).

Downloaded by Sky Park (parksky512@gmail.com)


lOMoARcPSD|32626796

Hinder Industries, which has developed two new products but has only enough plant capacity to introduce
one of these products this year: The company controller has gathered the following data to assist management in
deciding which product should be selected for production
Hinders fixed overhead includes proportional rent and utilities, machinery, depreciation, and supervisory
salaries. Selling and administrative expenses are not allocated to products
Cost per unit: Power Drill Power Saw
Raw materials P 22.00 P 18.00
Machining at P 12/hr. 9.00 7.50
Assembly at P 10/hr 15.00 5.00
Variable O/H at P8/hr 18.00 9.00
Fixed O/H at P4/hr 9.00 4.50
Total unit cost P 73.00 P 44.00
Suggested selling price P 88.98 P 49.95
Actual research and development costs P180,00 P 95,000
Proposed Advertising and promotion costs 300,000 250,000
9. For Hinder's power drill, the unit costs for raw materials. machining, and assembly represent
a. Conversion costs
b. Relevant costs
c. Committed costs
d. Prime costs

10. The difference between the P 49.95 selling price of Hinder's power saw and its total unit cost of P 44.00
represents the unit
a. Contribution margin ratio
b. Gross profit
c. Contribution
d. Residual income
11. The total overhead cost of P 13.50 for Hinder's power saw is
a. Carrying cost
b. Discretionary cost
c. Sunk cost
d. Mixed cost
12. Research and development cost for Hinders tv. to new products are
a. Conversion costs
b. Sunk costs
c. Relevant costs
d. Avoidable costs

13. The costs included in Hinder's fixed overhead are


a. Sunk costs
b. Discretionary costs
c. Committed costs
d. Opportunity costs

Downloaded by Sky Park (parksky512@gmail.com)


lOMoARcPSD|32626796

Borromeo Corp, who are reviewing the profitability of the company's four products and the potential effects of
several proposals for varying the product mix. An excerpt from the income statement and other data follow:

Each of the following proposals is to be considered independently of the other proposals. Consider only the
product changes stated in each proposal; the activity of other products remains stable. Ignore income taxes.

14. If product R is discontinued. the effect on income will be


a. P 4,194 increase
b. P 900 increase
c. P 1 ,368 Increase
d. P 12,600 increase

15. If product R is discontinued and a consequent loss of customers causes a decrease of 200 units in sales of Q,
the total effect on income will be
a. P 15, 600 decrease
b. P 1,250 decrease
c. P 2,044 increase
d. P 2,866 increase

16. If the sales of R is increased to P8 with a decrease in the number of units sold to 1,500, the effect on income
will be
a. P 2,199 decrease
b. P 600 decrease
c. P 750 increase
d. P 1,650 increase

17. The plant in which R is produced can be used to produce a new product, T. Total variable costs and expenses
per unit of T are P 8.05, and 1,600 units can be sold at P 9.50 each. If T is introduced and R is discontinued, the
total effect on income will be

Downloaded by Sky Park (parksky512@gmail.com)


lOMoARcPSD|32626796

a. P ,3220 increase
b. P 2,600 increase
c. P 2,320 increase
d. P 1 ,420 increase

18. Production of P can be doubled by adding a second shift, but higher wages must be paid, increasing the variable
cost of goods sold to P 3.50 for each additional unit. If the 1,000 additional units of P can be sold at P 10 each,
the total effect on income will
a. P 10,000 increase
b. P 6,500 increase
c. P 5,330 increase
d. P 2,260 increase

19. Part of the plant in which P is produced can easily be adapted to the production of S, but changes in quantities
may make changes in sales prices advisable. If production of P is reduced to 500 units (to be sold at P 12 each),
and production of S is increased to 2,500 units (to be sold at P 10.50 each), the total effect on income will be
a. P 2,060 decrease
b. P 1,515 decrease
c. P 250 increase
d. P 1,765 decrease

Downloaded by Sky Park (parksky512@gmail.com)


lOMoARcPSD|32626796

Wallace Corp., which sells Product A at a price of P 21 per unit. Wallace cost per unit based on the full capacity
of 200.000 units is as follows:
Direct materials P4
Direct labor 5
Overhead (2/3 of which is fixed) 6
P 15
A special order offering to buy 20,000 units was received from a foreign distributor. The only selling costs
that would be incurred on this order would be P 3 per unit for shipping. Wallace has sufficient existing capacity to
manufacture the additional units.

20. In negotiating a price of the special order, Wallace should set the minimum selling price per unit
a. P14
b. P15
c. P16
d. P18

21. To achieve an increase in operating income of P 40,000, Wallace should charge a selling price of
a. P14
b. P15
c. P16
d. P18

22. IM, Company has failed to reach its planned activity level during its first two years operation. The following
table shows the relationship between units produced, sales, and normal activity for these years and the projected
relationship for Year 3. All prices and costs have remained the same for the last two years and are expected to
do so in Year 3. Income has been positive in both Year 1 and Year 2.
Units Produced Sales Planned Activity
Year 1 90,000 90,000 100,000
Year 2 95,000 95,000 100,000
Year 3 90,000 90,000 100,000
Because IM Company uses an absorption costing system, one would predict gross margin for Year 3 to be
a. Greater than Year 1
b. Greater than Year 2
c. Equal to Year I
d. Equal to Year 2

23. A company had income of P50,000 using direct cost fora given period. Beginning and ending inventories for
that period were 13,000 units and 18,000 units, respectively. Ignoring income taxes, if the fixed overhead
application rate were P2.00 per unit, what would the income have been using absorption costing?
a. P 40,000
b. P50,000
c. P 60,000
d. Cannot be determined from the information given

Downloaded by Sky Park (parksky512@gmail.com)


lOMoARcPSD|32626796

24. A company has the following cost data:

Variable and absorption-cost net incomes are


a. P320 variable, P520 absorption
b. P330 variable, P590 absorption
c. P520 variable, P320 absorption.
d. P530 variable, P330 absorption.
25. Alcatraz Division of XYZ Corp. sells 80,000 units of part X to the outside market- Part X sells for P20, has a
variable cost of P 11 and a fixed cost per unit of P5 Alcatraz has a capacity to produce 100,000 units per
period. Capone Division currently purchases 10,000 units of part X from Alcatraz for P20. Capone has been
approached by an outside supplier willing to supply the parts for P18. What is the effect on XYZ's overall
profit if Alcatraz REFUSES the outside price and Capone decides to buy outside?
a. No change
b. P 70 ,000 decrease in XYZ profits
c. P 40,000 decrease in XYZ profits
d. P 20,000 increase in XYZ profits

26. Black Co.'s breakeven point was P780.OOO. Variable expenses averaged 60% of sales, and the margin of safety
was PI 30,000, What was Black's contribution margin?
a. P364,000
b. 546,000
c. 910,000
d. 1,300,000

27. Jackson Co. is considering a project that will use 2,000 square feet of storage space at one of its facilities to
store used equipment. What will determine Jackson's opportunity cost?
a. The net present value of the project
b. The internal rate of return of the project
c. The value of the next best use of the space
d. The depreciation expense on the space.

28. A recent college graduate has the choice of buying a new auto for P20,000 or Investing the money for four
years with a 6% expected annual rate of return, If the graduate decides tr purchase the auto, the best estimate of
the opportunity cost of that decision is:
a. l, 200
b. 4.800
c. 20,000
d. zero since there is no opportunity cost

Downloaded by Sky Park (parksky512@gmail.com)


lOMoARcPSD|32626796

29. Penny's TV and Appliance Store is a small company that has hired you to perform some management advisory
services. The following information pertains to 20x2 operations
Sales (2,000 TVs) 900,000
Cost of goods sold 400,000
Store manager's salary per year 70,000
Operating costs per year 157,000
Advertising and promotion per year 15,000
Commissions (4% of sales) 36,000
What are the estimates total costs if Penny expects to sell 3v000 units next year?
a. P896,000
b. P678,000
c. P 1,017,000
d. P799,000

30. Expenses that require a series of payments over a long period of time4such as long-term debt and lease
rentals4are frequently known as.
a. Programmed fixed expenses
b. Avoidable expenses
c. Variable expenses
d. Committed fixed expenses
e. e. normal capacity expenses

31. The coefficient of determination indicates-


a. Causal relationships among costs and other factors
b. The percentage of explained variance in the dependent variable
c. The linear relationship between two variables
d. Whether several variables fluctuate
e. The size of the standard deviation

32. When cost relationships are linear, total variable manufacturing costs will vary in proportion to changes in
a. machine hours
b. direct labor hours
c. total material cost
d. total overhead cost
e. volume of production
33. The term "relevant range" as used in cost accounting means the range over which
a. relevant costs are incurred
b. cost relationships are valid
c. costs may fluctuate
d. sates volume fluctuates
e. production may vary

Downloaded by Sky Park (parksky512@gmail.com)


lOMoARcPSD|32626796

34. The following relationships pertain to a year’s budgeted activity for Buckeye Company:
High Low
Direct labor hours 400,000 300,000
Total costs 154,000 129,000

What are the budgeted fixed cost for the year?


a. $ 100,000
b. $ 25,000
c. $ 54,000
d. $75,000
e. none of the above
35. When using conventional cost-volume-profit analysis, some assumptions about costs and sates prices are made.
Which of the following is one of those assumptions?
a. The contribution margin will change as volume increases
b. The variable cost per unit will decrease as volume increases
c. The sales price per unit will remain constant as volume increases
d. fixed cost per unit will remain the same as volume increases

36. Classifying a cost as fixed or variable depends on how it behaves


a. per unit, as the volume of activity changes
b. in total, as the volume of activity changes.
c. both A and B are correct
d. none of the above

37. Which of the following is a true statement about sales mix?


a. Profits may decline with an increase in total peso of sales if the sales mix shifts to sell more of the high
contribution margin product.
b. Profits may decline with an Increase in total peso of sales if the sales mix shifts to sell more of the lower
contribution margin product
c. Profits will remain constant with an increase in total peso of sales if the total sales in units remains constant.
d. Profits will remain constant with a decrease in total peso of sales if the sales mix also remains constant

38. The sates mix for Dial Enterprise is as follows:


Product A: 12 units @ P5.25 sales price, P4.85 variable cost per unit.
Product B: 10 units @ P7.50 sales price; P6.95 variable cost per unit.
Product C: 6 units @ P12.25 sales price; P10.35 variable cost per unit
Dial Enterprise's fixed costs are P75,950/
What are the composite break-even point?
a. 98,000
b. 2,000
c. 3,500
d. 4,000

Downloaded by Sky Park (parksky512@gmail.com)


lOMoARcPSD|32626796

39. The underlying difference between absorption costing and variable costing lies in the treatment of:
a. direct labor
b. variable manufacturing overhead
c. fixed manufacturing overhead
d. variable selling and administrative expenses
e. fixed selling and administrative expenses

40. In deciding whether or not to eliminate a branch or division, which of the following is considered relevant?
a. all variable costs of the branch
b. all direct costs of the branch
c. all fixed costs of the branch
d. all indirect costs of the branch
41. In analyzing whether to build another regional service office, the salary of the Chief Executive Officer (CEO)
at the corporate headquarters is:
a. Relevant because salaries are always relevant
b. Irrelevant since another imputed cost for the same will be considered
c. Relevant because this will probably change if the regional service office is built
d. Irrelevant because it is a future cost that will not differ between the alternatives under consideration

42. The salary you would otherwise earn by working rather than attending the CPA review is a good example of
a. a sunk cost
b. an opportunity cost
c. an Incremental cost
d. an out of pocket cost

43. The role of sunk costs in decision-making can be summed up in which of the following sayings?
a. no pain, no gain
b. penny saved is a penny earned
c. bygones are bygones
d. the love of money is the root of all evil
e. time is gold

44. When production exceeds sales, fixed manufacturing overhead costs


a. When production exceeds sales, fixed manufacturing overhead costs
b. Are deferred in inventory under absorption costing
c. Are released from inventory under variable costing
d. Are deferred in inventory under variable costing

Downloaded by Sky Park (parksky512@gmail.com)


lOMoARcPSD|32626796

45. Chris Company has fixed costs of P 100,000 and breakeven sales of P 800,000. What is its profit at
P1,200,000 sales?
a. P50,000
b. P200,000
c. P150,000
d. P400,000
46. Ube Company's variable costs are 75% of sales. At a sales level of P 400,000, the company's degree of
operating leverage is 8. At this level, fixed costs equal
a. P87,500
b. P50,000
c. P100,000
d. P75,000

Thompson Company is considering the development of two products: no. 65 or no. 66. Manufacturing cost
information follows.
No-65 No. 66
Annual fixed costs 220,000 340,000
Variable cost per unit 33 25

Regardless of which product is introduced, the anticipated selling price will be $50 and the company will pay a
10% sales commission on gross dollar sales. Thompson will not carry an inventory of these items.

47. What is the break-even sales volume (in dollars) on product no. 66? $850,000
48. Which of the two products will be more profitable at a sates level of 25,000 units? Product no. 66
49. At what unit-volume level will the profit/loss on product no. 65 equal the profit/loss on product no. 66?
15,000 units

The Bruggs & Strutton Company manufactures an engine for carpet cleaners called the "Snooper." Budgeted
cost and revenue data for the "Snooper are given below, based on sales of 40,000 units
Sales 1,600,000
Less: Cost of goods sold 1,120,000
Gross margin 480,000
Less: Operating expenses 100,000
Net income 380,000

Cost of goods sold consists of 800,000 of variable costs and $320,000 of fixed costs. Operating expenses consist of
$40,000 of variable costs and 60,000 of fixed costs

50. Calculate the safety margin. $800,000


51. Bruggs & Strutton received an order for 6,000 units at a price of S25.00. There will be no increase in fixed
costs, but variable costs will be reduced by $0.54 per unit because of cheaper packaging. Determine the
projected increase or decrease in profit from the order. $ 27,240

Downloaded by Sky Park (parksky512@gmail.com)


lOMoARcPSD|32626796

Wilcox Company is studying the impact of the following


1. An increase in sales price.
2. An increase in the variable cost per unit
3. An increase in the number of units sold (note: each unit produces a $6 contribution margin).
4. A decrease in fixed costs.
5. A proposed change in the method of compensation for sales people, away from commissions based on gross
sales dollars and toward higher monthly salaries.
Required
Determine the impact of each of these operating changes on Wilcox's per-unit contribution margin and break-even
point by completing the chart that follows. Your responses should be Increase (INC), Decrease (DEC), No Effect
(NE), or Insufficient Information to Judge (II).
1
52. Per-Unit Contribution Margin INC
Break-even Point DEC

2
53. Per-Unit Contribution Margin DEC
Break-even Point INC

3
54. Per-Unit Contribution Margin NE
Break-even Point NE

4
55. Per-Unit Contribution Margin NE
Break-even Point DEC

5
56. Per-Unit Contribution Margin INC
Break-even Point II

Boise Company manufactures and sells three products: Good, Better, and Best. Annual fixed costs are
$3,315,000 and data about the three products follow.
Good Better Best
Sales mix in units 30% 50% 20%
Selling price $250 $350 $500
Variable Cost 100 150 250

57. Determine the weighted-average unit contribution margin. $ 195


58. Determine the break-even volume in units for Good product. 5,100 units
59. Determine the total number of units that must be sold to obtain a profit for the company of $234,000
18200 units
60. Assume that the sales mix for Good, Better, and Best is changed to 50%, 30%, and 20%, respectively. Will
the number of units required to break-even increase or decrease? Increase

Downloaded by Sky Park (parksky512@gmail.com)


lOMoARcPSD|32626796

1. A company applies a variable overhead based on direct labor hours and has variable overhead
efficiency variance that $25,000 favorable A possible cause of this variance is:
a. Higher-skilled labor was used
b. Electricity rates were lower than expected
c. Fewer supplies were used than anticipated.
d. Fewer units of finished goods were purchased

2. For a given time period, a company had a favorable material quantity variance, a favorable
direct labor efficiency variance, and a favorable fixed overhead volume variance. Of the
following, the one factor that could not have caused all three variances is:
a. The purchase of higher quality materials
b. The use of lower skilled workers
c. The purchase of more efficient machinery
d. An increase in production supervision

3. A standard cost system can strengthen management cost control in all of the following ways
except:
a. Management will be able to manage by exception
b. Management will be able to set standards without input from production workers
c. The breakdown of variances into various components helps management trace the source
of potential cost problems
d. d Timely reporting of variances allows management to take corrective actions before costs
get out of hand

4. Highlight Inc, uses a standard cost system and applies factory overhead to products on the
basis of direct labor hours If the firm recently reported a favorable direct labor efficiency
variance, then:
a. The variable overhead spending variance must be favorable
b. The variable overhead efficiency variance must be favorable
c. The fixed overhead volume variance must be unfavorable
d. The direct labor variance must be unfavorable

5. A company has a raw material price variance that is unfavorable. An analysis of this variance
indicates that the company's only available supplier of one of its raw materials unexpectedly
raised the price of the material. The action management should take regarding this situation is:
a. Negatively evaluate the performance of the purchasing manager
b. Negatively evaluate the performance of the production manager
c. Change the raw material price standard
d. Ask the production manager to lower the material usage standard to compensate for
higher material costs

6. Richter Company has an unfavorable materials efficiency (usage) variance for a particular
month. Which of the following is least likely to be the cause of this variance?
a. Inadequate training of the direct labor employees.
b. Poor performance of the shipping employees
c. Poor design of the production process or product
d. Poor quality of the raw materials

7. Price variances and efficiency variances can be key to the performance measurement within a
company. In evaluating the performance within a company, a material efficiency variance can be
caused by all of the following except
a. Actions of the Purchasing Department
b. Design of the product
c. Skill level of the labor force
d. Sales volume of the product.

Downloaded by Sky Park (parksky512@gmail.com)


lOMoARcPSD|32626796

8. Fortune Corporation's marketing department recently accepted a rush order for a non-stock item
from a valued customer. The marketing department filed the necessary paperwork with the
production department, which complained greatly about the lack of time to do the job the right
way. Nevertheless, the production department accepted the manufacturing commitment and
filed the required paperwork with the purchasing department for the needed raw materials. A
purchasing clerk temporarily misplaced the paperwork. By the time the paperwork was found, it
was too late to order from the company's regular supplier. A new supplier was located, and that
vendor quoted a very attractive price. The materials arrived and were rushed into production,
bypassing the normal inspection processes (as directed by the production department supervisor)
to make up for lost time. Unfortunately, the goods were of low quality and created considerable
difficulty for Fortune's assembly line personnel. Which of the following best indicates the
responsibility for the materials usage variance in this situation?
a. Purchasing c. Marketing and production
b. Purchasing and marketing d. Purchasing marketing, and production\

9. Managerial accounting is similar to financial accounting in that


a. Both are governed by GAAP
b. Both deal with economic events
c. Both concentrate with historical costs
d. Both classify reported information in the same way

10. Which of the following is a controller's responsibility?


a. Tax planning and accounting c. In charge of credit and collections
b. Custodian of funds d. Arranging short-term financing

11. Water Control Inc manufactures water pumps and uses a standard cost system. The standard
factory overhead costs per water pump are based on direct labor hours and are shown below

Variable overhead (4 hours at $8/hour) $32


Fixed overhead (4 hours at $5/hour) 20
Total overhead cost per unit $52

*Based on a capacity of 100,000 direct labor hours per month.


The following additional information is available for the month of November.
• 22,000 pumps were produced, although 25,000 had been scheduled for production
• 94,000 direct labor hours were worked at a total cost of $940,000
• The standard direct labor rate is $9 per hour
• The standard direct labor time per unit is four hours
• Variable overhead costs were $740,000
• Fixed overhead costs were $540,000

The direct labor price variance for November was


a. $54,000 favorable c. $60,000 favorable
b. $94,000 unfavorable d. $40,000 unfavorable

12. Generally, management accounting involves the following functions except


a. Assists management in determining policies and in making plans
b. Assists management in tracing the taxes to be paid with the government
c. Assist management in the planning, directing, coordinating and controlling of operations
d. Maintain records and procedures which will be adequately protect all the interest related
to the business
13. Morisania Company produces a single product. The standard costs for one unit of its Mott
product are as follows.
Direct materials (6 pounds at $0.50 per pound) $3
Direct labor (2 hours at $10 per hour) 20
Variable manufacturing overhead (2 hours at $5 per hour) 10
Total $33

Downloaded by Sky Park (parksky512@gmail.com)


lOMoARcPSD|32626796

During August Year 2, 4,000 units of Mott were produced. The costs associated with August
operations were as follows:
Material purchased (36,000 pounds at $0.60 per pound) $21,600
Material used in production (28,000 pounds)
Direct labor (8,200 hours at $9.75 per hour) 79.950
Variable manufacturing overhead incurred 41,820

What is the labor rate variance for Mott for August Year 2?
a. $2,050 favorable
b. $2,000 unfavorable
c. $50 favorable
d. d $4,050 unfavorable

14. Microchips used in producing calculators is most likely a


a. Direct and Fixed
b. Direct and Variable
c. Indirect and Fixed
d. Indirect and Variable

15. Cost assignment is


a. Always arbitrary
b. Includes tracing and allocating
c. The same as cost accumulation
d. Finding the difference between budgeted and actual cost

16. Which statement is TRUE?


a. All variable costs are direct costs
b. Because of a cost-benefit tradeoff, some direct costs may be treated as indirect costs
c. All fixed costs are indirect costs
d. All direct costs are variable costs

17. Water Control Inc manufactures water pumps and uses a standard cost system. The standard
factory overhead costs per water pump are based on direct labor hours and are shown below

Variable overhead (4 hours at $8/hour) $32


Fixed overhead (4 hours at $5/hour) 20
Total overhead cost per unit $52

*Based on a capacity of 100,000 direct labor hours per month.


The following additional information is available for the month of November.

• 22,000 pumps were produced, although 25,000 had been scheduled for production
• 94,000 direct labor hours were worked at a total cost of $940,000
• The standard direct labor rate is $9 per hour
• The standard direct labor time per unit is four hours
• Variable overhead costs were $740,000
• Fixed overhead costs were $540,000

The direct labor efficiency variance for November was:

a. a $108,000 favorable
b. $60,000 favorable
c. $54,000 unfavorable
d. Od $60,000 unfavorable

18. Christopher Akers is the chief executive officer of SBL Inc. a masonry contractor. The financial
statements have just arrived showing a $3,000 loss on the new stadium job that was budgeted

Downloaded by Sky Park (parksky512@gmail.com)


lOMoARcPSD|32626796

to show a $6,000 profit. Actual and budget information relating to the materials for the job are
as follows:
Actual Budget
Bricks number of bundles 3,000 2,850
Bricks-cost per bundle $7.90 $8.00

Which one of the following is a correct statement regarding the stadium job for SBL?
a. The price variance was favorable by $285
b. The price variance was favorable by $300
c. The efficiency variance was unfavorable by $1,185
d. The flexible budget variance was unfavorable by $900

19. The controller for Durham Skates is reviewing the production cost report for July. An analysis of
direct material costs reflects an unfavorable flexible budget variance of $25. The plant manager
believes that this is excellent performance on a flexible budget for 5,000 units of direct material.
However the production supervisor is not pleased with this result, as he claims to have saved
$1,200 in material cost on actual production using 4.900 units of direct material. The standard
material cost is $12 per unit. Actual material used for the month amounted to $60,025.
If the direct material variance was investigated further it would reflect a price variance of
a. $850 unfavorable
b. $1,200 favorable
c. $1,225 unfavorable
d. $2.500 favorable

20. Nanjones Co. manufactures a line of products distributed nationally through wholesalers.
Presented below are planned manufacturing data for Year 2 and actual data for November Year
2. The company applies overhead based on planned machine hours using a predetermined annual
rate
Year 2 Planning Date
Annual November
Fixed manufacturing overhead $1,200,000 $100,000
Variable manufacturing overhead 2,400,000 220,000
Direct labor hours 48,000 4,000
Machine hours 240,000 22,000

Date for November Year 2

Direct labor hours (actual) 4,200


Direct labor hours (plan based on output) 4,000
Machine hours (actual) 21,600
Machine hours (plan based on output) 21,000
Fixed manufacturing overhead $101,000
Variable manufacturing overhead $214,000

The amount of over-or underapplied variable manufacturing overhead for November was

a. $6,000 overapplied c. $2,000 overapplied


b. $4,000 underapplied d. $6,000 underapplied

21. A company has $80,000 in common costs plus departments 1 and 2 have $40,000 and $60,000
in individual costs, respectively. The segments share a fleet of vehicles (the costs of which are
included in common costs) department 1 uses the vehicles for five months out of the year and
department 2 uses the vehicles for the remaining seven months. Using stand-alone cost
allocation, what is allocated to department 1?

Downloaded by Sky Park (parksky512@gmail.com)


lOMoARcPSD|32626796

a. $40,000 c. $66,667
b. $80,000 d. $33,333

22. Consider the following for Grideon Sporting Goods Manufacturing Co. for the prior year
• The company produced 1,500 units and sold 1,300 units both as budgeted.
• There were no beginning or ending work-in-process inventories and no beginning finished goods
inventory.
• Budgeted and actual fixed costs were equal, all variable manufacturing costs were affected by
production volume only, and all variable selling costs were affected by sales volume only.
• Budgeted per-unit revenues and costs were as follows:

Per Unit
Sales price $50
Direct materials $15
Direct labor $10
Other variable manufacturing costs $5
Fixed manufacturing costs $3
Variable selling costs $6
Fixed selling costs $2
Fixed administrative costs $1
The contribution margin earned by Gridiron Sporting Goods Manufacturing Co for the prior year
was:

a. $32,500 c. $18,200
b. 26,000 d. $10,400

23. Madtack Company's beginning and ending inventories for the month of November Year 1 are:

November 1 November 30
Direct materials $67,000 $62,000
Work-in-process 145,000 171,000
Finished goods 85,000 78,000
Production data for the month of November follows.

Direct labor $200,000


Actual factory overhead 132,000
Direct materials purchased 163,000
Transportation in 4,000
Purchase returns and allowances 2,000
Madtack uses one factory overhead control account and charges factory overhead to production
at 70 percent of direct labor cost. The company does not formally recognize over/underapplied
overhead until year-end. Madtack Company's cost of goods transferred to finished goods
inventory for November is:
a. 469,000 c. 484,000
b. 495,000 d. 476,000

24. The production manager of the Super T-shirt Co. is responsible for the activity of her
department and the costs associated with production. Super T-shirt adheres to a responsibility
center budget process, and the manager's performance is measured by how well she performs
to budget. Recently, the dark horse team won the local college basketball tournament. As a
result, the sales department, which operates as a profit center received an order for 10,000 t-
shirts, but only if they could be delivered in three days. The production manager said she could
meet the schedule, but only by incurring overtime pay that would cause her to be over budget
for hourly wages paid. What would be the best course of action for the sales department and
the production manager to undertake in this case?

Downloaded by Sky Park (parksky512@gmail.com)


lOMoARcPSD|32626796

a. Accept the order and charge the overtime to the production manager's budget.
b. Refuse the overtime and produce only what the production department is capable of while
staying within the budget.
c. Accept the order and ignore the effect on the production department budget when
conducting performance review.
d. Charge the overtime to the sales department's budget

25. The marketing manager of Ames Company has learned the following about a new product that
is being introduced by Ames. Sales of this product are planned at $100,000 for the first year.
Sales commission expense is budgeted at 8 percent of sales plus the marketing manager's
incentive budgeted at an additional 12 percent. The preparation of a product brochure will
require 20 hours of marketing salaried staff time at an average rate of $100 per hour, and 10
hours at $150 per hour, for an outside illustrator's effort. The variable marketing cost for the
new product will be:
a. 8,000 c. 10,000
b. 8,500 d. 10,500

26. An organization plans to implement a bonus plan based on segment performance. In addition,
the company plans to convert to a responsibility accounting system for segment reporting. The
following costs, which have been included in the segment performance reports that have been
prepared under the current system, are being reviewed to determine if they should be included
in the responsibility accounting segment reports.

I. Corporate CEO salary allocated on the basis of net segment sales.


II. Corporate training costs assigned on the basis of the number of employees in each
segment.
III. Fixed computer facility costs divided equally among each segment.
IV. Variable computer operational costs charged to each segment based on actual hours.
used times a predetermined standard rate; any variable cost efficiency or inefficiency
remains in the computer department

Of the four cost items, the item that would most logically be included in the segment
performance reports prepared on a responsibility accounting basis would be the:

a. Fixed computer facility costs


b. Corporate administrative costs
c. Training costs
d. Variable computer operational costs

27. XYZ Co. has two business units selling two different products. Business unit A produces and sells
sprinkler heads and requires customers to pay for their purchase prior to the production of the
sprinkler heads. Business unit A recognizes revenue at the time of payment. Business unit B
produces and sells household faucets and allows customers to pay for the faucets after they are
delivered to the customer. Business unit 8 recognizes revenue when the customer receives the
product. What is the likely effect that will occur by management in measuring and comparing
the performance of each business unit?

a. Business unit A will have overstated revenue in comparison to business unit B


b. Business unit A and business unit B can be effectively compared
c. Business unit A will have understated revenue in comparison business unit B.
d. Management cannot effectively compare the business units.

28. Parkside Inc. has several divisions that operate as decentralized profit centers. Parkside's
Entertainment Division manufactures video arcade equipment using the product of two of

Downloaded by Sky Park (parksky512@gmail.com)


lOMoARcPSD|32626796

Parkside's other divisions. The Plastics Division manufactures plastic components, one type that
is made exclusively for the Entertainment Division, while other less-complex components are
sold to outside markets. The products of the Video Cards Division are sold in a competitive
market; however, one video card model is also used by the Entertainment Division. The actual
costs per unit used by the Entertainment Division are presented below:

Plastic Components Video Cards


Direct material $1.25 $2.40
Direct labor 2.35 3.00
Variable overhead 1.00 1.50
Fixed overhead .40 2.25
Total cost $5.00 $9.15
The Plastics Division sells its commercial products at full cost plus 25 percent markup and
believes that the proprietary plastic component made for the Entertainment Division would sell
for $6.25 per unit on the open market. The market price of the video card used by the
Entertainment Division is $10.98 per unit.

A per-unit transfer price from the Video Cards Division to the Entertainment Division at a full
cost of $9.15 would:

a. Allow evaluation of both divisions on a competitive basis


b. Demotivate the Entertainment Division and cause mediocre performance
c. Provide no profit incentive for the Video Cards Division to control or reduce costs
d. Encourage the Entertainment Division to purchase video cards from an outside source.

29. An appropriate transfer price between two divisions of Speaker's Warehouse can be determined
from the following data:
Fabricating Division
Market price of subassembly $50
Variable cost of subassembly $25
Excess capacity (in units) 40
Assembling Division
Number of units needed 30

What is the natural bargaining range for the two divisions?

a. Any amount between $50 and $75


b. Any amount less than $50
c. Any amount between $25 and 550
d. d Any amount over $50

30. Troughton Company manufactures radio-controlled toy dogs. Summary budget financial data for
Troughton for the current year are as follows:

Sales (5,000 units at $150 each) $750,000


Variable manufacturing cost 400,000
Fixed manufacturing cost 100,000
Variable selling and administrative cost 80,000
Fixed selling and administrative cost 150,000

Troughton uses an absorption costing system with overhead applied based on the number of
units produced, with a denominator level of activity of 5,000 units. Underapplied or overapplied
manufacturing overhead is written off to cost of goods sold in the year incurred. The $20,000
budgeted operating income from producing and selling 5,000 toy dogs planned for this year is of
concern to Trudy George, Troughton's president. She believes she could increase operating to
$50,000 (her bonus threshold) if Troughton produces more units than it sells, thus building up

Downloaded by Sky Park (parksky512@gmail.com)


lOMoARcPSD|32626796

the finished goods inventory. How much of an increase in the number of units in the finished
goods inventory would be needed to generate the $50,000 budgeted operating income?

a. 556 units c. 1,500 units


b. 600 units d. 7,500 units

31. Parkside Inc. has several divisions that operate as decentralized profit centers. Parkside's
Entertainment Division manufactures video arcade equipment using the product of two of
Parkside's other divisions. The Plastics Division manufactures plastic components, one type that
is made exclusively for the Entertainment Division, while other less-complex components are
sold to outside markets. The products of the Video Cards Division are sold in a competitive
market; however, one video card model is also used by the Entertainment Division. The actual
costs per unit used by the Entertainment Division are presented below:

Plastic Components Video Cards


Direct material $1.25 $2.40
Direct labor 2.35 3.00
Variable overhead 1.00 1.50
Fixed overhead .40 2.25
Total cost $5.00 $9.15
The Plastics Division sells its commercial products at full cost plus 25 percent markup and
believes that the proprietary plastic component made for the Entertainment Division would sell
for $6.25 per unit on the open market. The market price of the video card used by the
Entertainment Division is $10.98 per unit.

A per-unit transfer price from the Video Cards Division to the Entertainment Division at a full
cost of $9.15 would:

a. Allow evaluation of both divisions on a competitive basis


b. Encourage the Entertainment Division to purchase video cards from an outside source
c. Demotivate the Entertainment Division and cause mediocre performance
d. Provide no profit incentive for the Video Cards Division to control or reduce costs

32. Natural Lumber Co has two divisions. The Forestry Division cuts down trees and sells them to
the Sawing Division. The Sawing Division cuts the lumber into boards that are sold to external
customers for further manufacturing into building products. The costs and market prices for the
two divisions are listed below. Each tree on average can be made into 25 boards.

Division Amounts
Forestry Division
Variable cost per tree $400
Fixed cost per tree 800
Desirable full cost markup % 8%
Sawing Division
Variable cost per board $10
Fixed cost per board 30
Market price per tree supplied (external supplier) 1,300
Market price per board sold (external customer) 100
Market prices can fluctuate slightly and the market is moderately competitive. All of the trees
harvested by the Forestry Division can be sold to an external customer by at least the full cost.
What is the best range of negotiated transfer prices for each tree from the Forestry Division to
the Sawing Division, assuming that each division has the incentive to improve its own
profitability?

a. $1,296 to $1,300 c. $432 to $1,296


b. $1,200 to $1,300 d. $400 to $1,200

Downloaded by Sky Park (parksky512@gmail.com)


lOMoARcPSD|32626796

33. Kern Manufacturing has several divisions and evaluates performance using segment income.
Because sales include transfers to other divisions, Kern has established a price for internal sales
as cost plus 10 percent. Red Division has requested 10,000 units of Green Division's product
Green Division is selling its product externally at a 60 percent markup over cost. The corporate
policy will encourage the Green Division to:
a. Transfer the product to the Red Division because all costs are being covered and the
division will earn a 10 percent profit
b. Reject the sale to the Red Division because it does not provide the same markup as
external sales
c. Accept the sale to the Red Division if its operating a full capacity and the sale will
contribute to the fixed costs
d. Transfer the product to the Red Division if it does not require the Green Division to give up
any external sales

34. Manhattan Corp has several divisions that operate as decentralized profit centers At the present
time, the Fabrication division has excess capacity of 5,000 units with respect to the UT-371
circuit board, a popular item in many digital applications. Information about the circuit board
follows
Market price $48
Variable selling/distribution costs on external sales 5
Variable manufacturing cost 21
Fixed manufacturing cost 10

Manhattan's Electronic Assembly division wants to purchase 4,500 circuit boards either internally,
or else use a similar board in the marketplace that sells for $46. The Electronic Assembly division's
management feels that if the first alternative is pursued, a price concession is justified, given that
both divisions are part of the same firm. To optimize the overall goals of Manhattan, the minimum
price to be charged for the board from the Fabrication division to the Electronic Assembly division
should be:
a. $21 c. $31
b. $26 d. $46

35. Which one of the following is the best reason for using variable costing?
a. Fixed overhead is more closely related to the capacity to produce than to the production
of specific units.
b. All costs are variable in the long term.
c. Variable costing is acceptable for income tax purposes.
d. Variable costing usually results in higher operating income than absorption costing.

36. A master budget for the year shows anticipated sales of 1,000 units, a sale price of $25 per unit,
variable costs of $15 per unit, and fixed costs of $6,000. Actual sales for the year are 1,200 units
and actual operating income for the year is $4,250. Relative to expected results using flexible
budgeting actual operating income was:
a. Higher by $250 c. Lower by $550
b. Higher by $2,000 d. Lower by $1,750

37. A retailer sells a product throughout its stores for $30 for each unit. Fixed costs include rent of
$60,000, salaries of $200,000 and other fixed costs of $100,000. Each unit’s whole cost is
$16.50. The salesperson receives a 5 percent sales commission in addition to the fixed salary
that the salesperson receives. If the company expects to sell 35,000 units next year, what is the
estimated operating income?
a. $112,500 c. $0
b. $60,000 d. ($60,000)

Downloaded by Sky Park (parksky512@gmail.com)


lOMoARcPSD|32626796

38. Vivienne Manufacturing Co produces basketballs. The following information relates to the
production and sale of basketballs for the year.
Sales price per unit $95
Variable production costs per unit $35
Variable selling & admin costs per unit $10
Fixed overhead costs for year $200,000
Fixed selling & admin costs for year $65,000
Units produced and sold this year 7,500 units
Assuming variable costing, what is the amount reported on the income statement for variable
manufacturing cost of goods sold?
a. 262,500 c. 462,500
b. 337,500 d. 727,500

39. A detergent company sells large containers of industrial cleaner at a selling price of $12 per
container. Each container of cleaner requires $4.50 of direct materials, $2.50 direct labor, and
$1 of variable overhead. The company has total fixed costs of $2,000,000 and an income tax rate
of 40 percent. Management has set a goal to achieve a targeted after-tax net income of
$2,400,000. What amount of dollar sales must the company achieve in order to meet its goal?
a. $14,400,000 c. $22,000,000
b. $18,000,000 d. $24,000,000

40. Almo Company developed its business plan based on the assumption that canopies would sell at
a price of $400 each. The variable costs for each canopy were projected at $200, and the annual
fixed costs were budgeted at $100,000. Alamo's after-tax profit objective was $240,000, the
company's effective tax rate is 40 percent. If no changes are made to the selling price or cost
structure, determine the number of units that Almo Company must sell to achieve its after-tax
profit objective.
a. 1,700 units c. 3,500 units
b. 2,500 units d. 4,500 units

41. Delphi Co. has developed a new product that will be marketed for the first time during the next
fiscal year. Although the marketing department estimates that 35,000 units could be sold at $36
per unit, Delphi's management has allocated only enough manufacturing capacity to produce a
maximum of 25,000 units of the new product annually. The fixed costs associated with the new
product are budgeted at $450,000 for the year, which includes $60,000 for depreciation on new
manufacturing equipment Data associated with each unit of product are presented below
Delphi is subject to a 40 percent income tax rate.

Variable costs
Direct material $7.00
Direct labor 3.50
Manufacturing overhead 4.00
Total variable manufacturing cost 14.50
Selling expenses 1.50
Total variable cost $16.00
The number of units of the new product that Delphi Co. must sell during the next fiscal year in
order to break even is

a. 20,930 c. 22,500
b. 21,300 d. 19,500

Downloaded by Sky Park (parksky512@gmail.com)


lOMoARcPSD|32626796

42. Bruell Electronics Co is developing a new product, surge protectors for high-voltage electrical
flows. The following cost information relates to the product
Unit costs
Direct materials $3.25
Direct labor 4.00
Distribution 0.75
The company will also be absorbing $120,000 of additional fixed costs associated with this new
product. A corporate fixed charge of $20,000 currently absorbed by other products will be
allocated to this new product.

How many surge protectors (rounded to the nearest hundred) must Bruell Electronics sell at a
selling price of $14 per unit to increase after-tax income by $30,000? Bruell Electronics effective
income tax rate is 40 percent.

a. 15,000 units c. 25,000 units


b. 20,000 units d. 28,300 units

43. Alex Co had the following inventories at the beginning and the end of the month of January

January 1 January 31
Finished goods $125,000 $117,000
Work-in-process 235,000 251,000
Direct materials 134,000 124,000
The following additional manufacturing data were available for the month of January
Direct materials purchased $189,000
Purchase returns and allowances 1,000
Transportation in 3,000
Direct labor 300,000
Actual factory overhead 175,000
Alex Co uses normal costing to determine inventory cost and applies factory overhead at a rate
of 60 percent of direct labor cost.

Alex Co's total manufacturing cost for January was

a. 681,000 c. 671,000
b. 669,000 d. 679,000

44. Phillips and Company produces educational software. Its unit cost structure, based upon an
anticipated production volume of 150 000 units, is as follows

Sales price $160


Variable costs 60
Fixed costs 55

The marketing department has estimated sales for the coming year at 175,000 units, which is
within the relevant range of Phillip's cost structure. Phillip's breakeven volume (in units) and
anticipated operating income for the coming year would amount to:

a. 82, 500 units and $7,875,000 of operating income.


b. 82,500 units and $9,250,000 of operating income
c. 96,250 units and $3,543,750 of operating income
d. 96,250 units and $7,875,000 of operating income

Downloaded by Sky Park (parksky512@gmail.com)


lOMoARcPSD|32626796

45. Folsom Fashions sells a line of women's dresses Folsom's performance report for November
Year 1 follows

Actual Budget
Dresses sold 5,000 6,000
Sales $235,000 $300,000
Variable costs 145,000 180,000
Contribution margin 90,000 120,000
Fixed costs 84,000 80,000
Operating income $ 6,000 $40,000

The company uses a flexible budget to analyze its performance and to measure the effect on
operating income of the various factors affecting the difference between budgeted and actual
operating income.

The sales price variance for November is.

a. 30,000 unfavorable c. $20,000 unfavorable


b. $18,000 unfavorable d. $15,0000 unfavorable

46. Profits that are lost by moving an input from one use to another are referred to as
a. Out of Pocket Costs c. Replacement Costs
b. Cannibalization charges d. Opportunity Costs

47. Gardener Company currently is using its full capacity of 25,000 machine hours to manufacture
product XR-2000. LJB Corporation placed an order with Gardener for the manufacture of 1,000
units of KT-6500. LJB would normally manufacture this component. However, due to a fire at its
plant, LJB needs to purchase these units to continue manufacturing other products. This is a
one-time special order. The following reflects unit cost data and selling prices

KT-6500 XR-2000
Material $27 $24
Direct labor 12 10
Variable overhead 6 5
Fixed overhead 48 40
Variable selling and administrative 5 4
Fixed selling and administrative 12 10
Normal selling price $125 $105
Machine hours required 3 4
What is the minimum unit price that Gardener should charge LJB to manufacture 1,000 units of
KT- 6500?

a. 93 c.110
b. 96.5 d. 125

48. Reynolds Inc has three divisions, one of which has been underperforming its peers for the past
several years. The underperforming division has sales of $480,000, with variable costs of
$350,000. Although fixed costs total $210,000 for the division, Reynolds anticipates that $95,000
of those costs will still remain even if the division is dropped. Reynolds should
a. Drop the division because total cost exceeds total sales
b. Keep the division because the CM exceeds unavoidable fixed costs
c. Drop the division because the avoidable fixed costs exceed the CM
d. Keep the division because the CM exceeds the avoidable fixed costs

Downloaded by Sky Park (parksky512@gmail.com)


lOMoARcPSD|32626796

49. The Danforth Corp. circuit production plant has a 12,000-unit capacity and currently produces
10,000 circuits per year. The company incurs $50,000 in variable costs for its current production
and carries a $40,000 fixed cost burden. If Danforth has an opportunity to fill a special order for
1,000 circuits, the price per unit for the order should exceed
a. 4 c. 8.33
b. 5 d. 9

50. Which of the following is not a qualitative factor that Atlas Manufacturing should consider when
deciding whether to buy or make a part used in manufacturing their product?
a. Quality of the outside producer's product
b. Potential loss of trade secrets
c. Manufacturing deadlines and special orders
d. Variable cost per unit of the product.

Downloaded by Sky Park (parksky512@gmail.com)


lOMoARcPSD|32626796

The term relevant cost applies to all the following decision situations except the
A. Acceptance of a special order. ✓
B. Determination of a product price.
C. Replacement of equipment. ✓
D. Addition or deletion of a product line. ✓

Explanation:
Relevant costs are those used in making decision- applied in a single decision Includes variable cost. These
costs have two characteristics – differential costs and future costs. The term relevant costs applies to the
acceptance or rejection of a special sales order, replacement or retention of equipment, addition or deletion of a
product line, and even in the determination of a product price. Among the choices given, however, choice-letter <b=
is the best answer because relevant cost is least applied in the determination of regular selling price. Full absorption
cost is used in the determination of product price.

Decision Criteria in Relevant Costing


Choose the option that has the lower cost. In most cases,
Make or buy a part/product
fixed costs are irrelevant. Consider opportunity costs, if any.
Accept the order when the additional revenue from the special order
Accept or reject a special order exceeds additional cost, provided the regular market will not be
affected. In most cases, fixed costs are irrelevant
Continue if segment’s avoidable revenue > avoidable costs; otherwise
Continue or shutdown a
consider shutting down the segment since allocated fixed cost is
business segment
usually unavoidable, it is considered irrelevant.
Process further if additional revenue from processing further is greater
than further processing costs.
Sell or process further a
product
Joint costs, since already incurred prior to the split-off point, are
considered sunk costs and irrelevant.
Best product combination Identify and measure the constraint on the limited resources. Rank the
(optimization of scarce products according to the highest contribution margin per unit of limited
resources) resources
Identify the factor to change and the amount of contemplated change.
Change in profit factors
Change the profit factor if it will cause an improvement on the
(related with CVP analysis)
company’s overall profit position.

Management accountants are frequently asked to analyze various decision situations including the following
I. The cost of a special device that is necessary if a special order is accepted.
II. The cost proposed annually for the plant service for the grounds at corporate headquarters.
III. Joint production cost incurred to be considered in a sell-at-split versus a process further decision.
IV. The cost of alternative use of plant space to be considered in a make-or-buy decision.
V. The cost of obsolete inventory acquired several years ago, to be considered in a keep-versus-disposal
decision.
The cost described in situations I and IV are
A. Prime cost
B. Discretionary costs
C. Relevant costs
D. Differential costs

Explanation: DIFFERENTIATE RELEVANT AND DIFFERENTIAL


Choice-letter <c= is the best answer. Item I, the cost of a special device that is necessary if a special
order is accepted is an incremental cost of accepting the order and is a relevant cost. Item IV, the cost of
alternative use of plant space to be considered in a make-or-buy decision, is an opportunity cost and, is
relevant in making decision.
Choice a is incorrect. Prime costs include direct material and direct labor.
Choice "b" is incorrect. Discretionary costs are discretionary. In I, the special device is necessary.
Choice "c" is incorrect. Differential cost is the difference between the cost of two alternative decisions, or of a
change in output levels.

Downloaded by Sky Park (parksky512@gmail.com)


lOMoARcPSD|32626796

The costs described in situations III and V are


A. Prime costs
B. Sunk costs
C. Discretionary costs
D. Relevant costs

Explanation:
Item III, joint production cost already incurred, and item V would no longer be avoided whether the
products are sold at split-off point or not. The cost of obsolete inventory acquired several years ago in relation
to the decision whether to keep or dispose the inventory is already incurred, a sunk cost. In both situations,
the costs described are sunk costs and are irrelevant in decision-making. Hence, choice-letter <b= is correct.

The cost described in II is a


A. Prime costs
B. Discretionary costs – not essential
C. Relevant costs
D. Differential costs
Choice "b" is correct. Discretionary cost. The proposed cost for plant service for the grounds at corporate
headquarters is an example of an avoidable cost that is discretionary.
Choice "a" is incorrect. Prime costs are direct materials and direct labor.
Choice "c" is incorrect because relevant costs are determined when decisions are to be made, and not
merely a proposal, which may be decided to be incurred or not.
Choice "d" is incorrect. Differential cost is t is a quality of a relevant cost which is determinable only at a time
a decision is to be made.

Opportunity costs are


A. Costs irrevocably incurred by past actions.
B. The difference between actual and standard costs.
C. Not recorded in the accounting records.
D. Partly fixed costs and partly variable costs.

Explanation: Opportunity costs are those benefits foregone in favor of the alternative chosen. They are
sacrificed benefits, not incurred but is implied, and not recorded in the accounting books because they are
theoretical. Hence, choice-letter <c= is correct.
Choice-letter <a= is incorrect because costs incurred by past actions are past cost or sunk costs.
Choice-letter <b= is incorrect because the difference between actual and standard cost is called a cost variance.
Choice-letter <d= is incorrect because costs that are partly fixed and partly variable are called mixed costs

Cost of goods sold is a component of the income statement. In a merchandising establishment, this refers to purchases
adjusted for changes in inventory. In a manufacturing company, what replaced purchases to arrive at cost of goods
sold?
A. Finished goods
B. Fixed manufacturing overhead
C. Work in process inventory
D. Cost of goods manufactured

Explanation:
Choice-letter <d= is correct. In the computation of cost of goods sold, net purchases are replaced by
the cost of goods manufactured (COGM) in a manufacturing company. Just like merchandise purchases, CGM
is added to finished goods-beginning and then finished goods-ending is deducted to get the cost of goods
sold.
Choice-letter <a= is incorrect because finished goods inventories are added to or deducted from
COGM to arrive at COGS.
Choice-letter <b= is incorrect because fixed manufacturing overhead is directly used in the
computation of total factory costs.
Choice-letter <c= is incorrect because work-in-process inventory is used in the computation
of COGM, not in COGS

Downloaded by Sky Park (parksky512@gmail.com)


lOMoARcPSD|32626796

When all manufacturing cost used in production are attached to the products, whether direct, or indirect, variable or
fixed, this is called
A. Process costing
B. Absorption costing
C. Variable costing
D. Job order costing

Explanation:
Choice-letter <b=, absorption costing, is the correct answer. Absorption costing includes all
manufacturing costs whether direct or indirect, variable or fixed, in the determination of product costs. This is
the traditional concept of cost determination, is in accordance with the generally accepted accounting
principles, and includes all manufacturing costs as inventoriable costs because they are all relevant
expenditures in the manufacturing process.
Choice-letter <a=, process costing, is incorrect because it relates to the model of accumulating
production costs by following the departmentalized transfer of units in the production process. Choice-letter
<c=, variable costing, is incorrect because it assigns only variable production cost as product cost. Choice-
letter <d=, job order costing, is incorrect because it refers to the model of accumulating production per order
made by customers.
Al-kris Company uses a regression equation to analyze the behavior of its transportation costs (T) as a function of travel
time (H). They developed the following equation using two years’ observation with a related coefficient of determination
of 85: T= 100,000 + P50H

If 500 hours of travel time were logged in one period, the related point estimate of total transportation costs would be
A. P110,000
B. P121,250
C. P106,250
D. P125,000

Explanation: Total Cost= Fixed Cost + Variable Cost


Fixed Cost 100,000
Variable Cost 25,000 (P 50(500H)
Total Cost P125,000

These are among the methods of segregating fixed cost and variable costs except
A. Breakeven method
B. Simple regression analysis
C. Scattergraph method
D. High-low method.

Explanation:
Choice-letter <a= is the correct choice. The breakeven method is not used in segregating fixed into
variable costs but is instead used in analyzing the potential profitability and areas of sensitivity in controlling
profit.
The methods used in separating the fixed and variable costs elements are: practical method (e.g.,
historical records), simple average high-low method, weighted average high-low method, scattergraph
method, and least-squares method.

The segregation of fixed costs and variable costs is key to proper cost analysis. Regression analysis is a technique
used for this purpose. Identify the appropriate statements below on regression analysis:
1. It assumes that a change in value of a dependent variable (y or total cost) is related to the change in the
value of an independent variable (x or cost driver).
2. A linear relationship between direct cost and production volume can cause a problem when using
accounting data for regression analysis. (doesn’t cause a problem)
3. It attempts to find an equation for the linear relationship among variable. (just establishes a relationship
among the variables)
4. It establishes a cause and effect relationship.

Downloaded by Sky Park (parksky512@gmail.com)


lOMoARcPSD|32626796

A. All four statements are appropriate.


B. Statements 1, 3 and 4 only.
C. Statements 1 and 3 only.
D. Statements 2 and 4 only.

Explanation:
Statements 1 and 3 are correct with respect to regression analysis. Regression analysis formulates
a linear equation with the end-in-view of estimating the value of <Y=, the dependent variable.
Statement 2 is false because a linear relationship between direct cost and production volume does
not cause a problem but rather complement accounting data for regression analysis. Statement 4 is also false
because a regression line does not establish a cause and affect relationship but rather a correlation between
independent and dependent variables.

For the month just ended, the cost components to make Product AB was P50 per unit plus fixed costs of P250,000.
One thousand units were produced. For the month the cost to make the product will be P55 per unit plus fixed cost of
P250,000. Fifteen hundred units are expected to be produced. The estimates of the underlying but unknown intercept
and slope coefficient for the current month are
A. P250,000 and P50
B. P55 and P250,000
C. PP50 and P250,000
D. P250,000 and P55

Explanation:
The intercept or point of origin is the fixed costs; while the slope coefficient is the variable cost rate. The fixed
costs for the current month is P250,000 and the variable cost rate is P55 per unit.

In preparing the annual profit plan for the coming year, Venus Company wants to determine the cost behavior pattern
of the maintenance costs. Venus has decided to use linear regression by employing the equation Y = a + bx for
maintenance costs. The prior year’s data regarding maintenance hours and costs and the results of the regression
analysis are given below,
Average cost per hour P 9.00
a 684.65
b 7.2884
Standard error of a 49.515
Standard error of b .12126
Standard error of the estimate 34.469
r2 .99724

In the standard regression equation Y = a + bx, the letter b is best described as a(n):
A. Independent variable
B. Dependent variable
C. Constant coefficient
D. Variable coefficient

The letter <x= in the standard regression equation is best described as a (an)
A. Independent variable
B. Dependent variable
C. Constant coefficient
D. Coefficient of determination

Based upon the data described from the regression analysis, 420 maintenance hours in a month would mean the
maintenance costs (rounded to the nearest peso) would be budgeted at
A. P3,780
B. P3,600
C. P3,790
D. P3,746

Downloaded by Sky Park (parksky512@gmail.com)


lOMoARcPSD|32626796

Explanation:
X= 420 hours
Y= a + bx
Y= 684.65 + 7.2884(420)
Y= 684.65 + 3061
Y= 3745.65 → 3746

Pure Company has developed a regression equation to analyze the behavior of its maintenance costs (Q) as a
function of machine hours (Z). The following equation was developed by using 30 monthly observations with a related
coefficient of determination of 0.90: Q = P6,000 + P5.25Z
If 1,000 machine hours are worked in one month, the related point of estimate of total maintenance costs would be
A. P11,250
B. P10,125
C. P 5,250
D. P 4,725

Explanation:
Z= 1,000 machine hours
Q= 6,000 + P5.25(1000)
Q= 6,000 + 5,250
Q= 11,250

Y =P575,000 + P8.50x represents the behavior of maintenance costs (Y) as a function of machine hours (x). Thirty (30)
monthly observations were used to develop the foregoing regression equation. The related coefficient of determination
was .90. If 2,500 machine hours were worked in one month, the related point estimate of total variable maintenance
costs would be:
A. P23,000
B. P21,250
C. P25,250
D. P19,125

Explanation: x= 2,500 hours


Y= 575,000 + P8.50(x)
8.50(2,500) = Estimated Variable MC
21,250 =

Dongian, Inc. is preparing a flexible budget for the next year and requires a breakdown of the cost of steam used in its
factory into the fixed and variable elements. The following data on the cost of steam used and direct labor hours worked
are available for the last 6 months of this year.

Month Cost of Steam Direct Labor Hours


July P15,850 3,000
August 13,400 2,050
September 16,370 2,900
October 19,800 3,650
November 17,600 2,670
December 18,500 2,650
P101,520 16,920

Assuming that Dongian uses the high-low method of analysis. The estimated variable cost of steam per direct labor
hour is:
A. P4.00
B. P5.42
C. P5.82
D. P6.00

Downloaded by Sky Park (parksky512@gmail.com)


lOMoARcPSD|32626796

Explanation:
The high-low method relates to range analysis where in a range there is the highest point and the
lowest point. The high-low method identifies the highest observation and the lowest observation and getting
their differences both in units and in amount. The highest observation is the month of October while the lowest
observation is the month of August. Tabulating the data of the highest and the lowest occurrences, we have:

Direct labor hours Cost


Highest (October) 3,650 19,800
Lowest (August) 2,050 13,400
Difference 1,600 P 6,400

VCR =  in Costs /  in Units = P6,400 / 1,600 = P4 / DLH

Under the HLM, the variable cost rate is equal to the change in costs over the change in units. This
high-low method of computing the variable cost per unit rests on the principle that a change in cost is attributed
to variable, since total fixed cost remains unchanged.
The HLM is used when there is a positive relationship between costs and units. That is, if total unit
increases, total cost also increases and if total unit decreases, total cost also decreases. Otherwise, HLM is
not applicable, and the scattergraph method and least squares method are to be used in segregating the
variable costs from the fixed costs

What is the amount of fixed cost?


A. P14,600
B. P 8,200
C. P 5,200
D. P 0

The total production cost for 20,000 units was P21,000 and the total production cost for making 50,000 units was
P34,000. Once production exceeds 25,000 units, additional fixed costs of P4,000 were incurred. The full production
cost per unit for making 30,000 units is:
A. P 0.30
B. P 0.68
C. P 0.84
D. P0.93

Explanation:
The full production cost per unit is composed of the unit variable cost and unit fixed cost. The unit variable
cost is determined as follows:
Unit variable cost= = (Δ in Costs – Increase in Fixed costs) / Δ in Sales
= [(P34,000 – P21,000) – P4,000] / (50,000 – 20,000)
= P9,000 / 30,000 = P0.30
The total fixed costs should be determined at 50,000 units since the cost of per unit to be computed
is at 30,000 units. At 50,000 units the increase in fixed costs of P4,000 is already included. Fixed cost is total
cost less variable cost. Therefore, the total fixed cost is P19,000 [i.e., P34,000 – (50,000 x P0.30)]. Finally,
the unit cost at 30,000 units shall be:

Unit variable costs (constant) P 0.30


Unit fixed costs (P19,000/30,000 units) 0.63
Unit cost (at 30,000 units) P 0.93

Downloaded by Sky Park (parksky512@gmail.com)


lOMoARcPSD|32626796

For the six months of the year, the highest level of activity for MDG Corporation was 18,000 full units of production with
maintenance cost at P114,000 and its lowest level of activity for the same period was at 14,000 full units of production
with maintenance cost at P94,000. What amount of maintenance cost should MDG expect in a month in which it was
scheduled 16,000 equivalent full units of production?
A. P 24,000
B. P104,000
C. P 80,000
D. P114,000

Explanation:
The amount of maintenance cost is composed of fixed costs and variable costs. The problem gives
two levels of activities with their corresponding total costs. The variable cost rate could be determined using
the high-low method of costs segregation. Once the variable cost rate is determined, the total fixed costs is
computed by deducting the variable costs from the total fixed costs.

Using the high-low method (HLM), we have:


Units Total Costs Variable Cost* Foxed Cost **
High 18,000 114,000 90,000 24,000
Low 14,000 94,000 70,000 24000
Difference 4000 20,000

* Var. Costs (18,000 units x P5) = P90,000


* Var. Costs (14,000 units x P5) = 70,000
** Fixed Costs = Total Costs - Variable Costs

The variable cost rate (VCR) is:


VCR = Change in total costs / Change in units
= P20,000 / 4,000 units = P5.00/ unit

Therefore, the total maintenance costs at 16,000 units is:


Variable costs (16,000 units x P5) P 80,000
Fixed costs 24,000
Total costs (at 16,000 units) P104,000

Simple regression analysis provides the means to evaluate a line of regression, which is fitted to a plot of data and
represents
A. The way costs change with respect to the dependent variable.
B. The way costs change with respect to both independent variable and dependent variables.
C. The variability expense with pesos of production.
D. The way costs change with respect to the independent variable.

Explanation:
A simple regression line represents the way the dependent variable (<Y=, normally
described in pesos) varies with respect to the changes in the independent variable (<x=, normally
presented in units).

Downloaded by Sky Park (parksky512@gmail.com)


lOMoARcPSD|32626796

Choice-letters <a= and <b= are incorrect because costs change with respect to independent
variable, not with dependent variable or both. Choice-letter <c= is incorrect because the variability of
expenses is related to units of measurement and not normally in terms of pesos of production.

The slope of the line of regression is


A. The rate at which the independent variable varies.
B. The rate at which the dependent variable varies.
C. The level of fixed costs.
D. The level of the total variable costs.

Explanation:
The slope is the <b= in the regression line Y = a + bx. Applying this equation to costs, we
have: Total costs = fixed costs + variable costs.
The <b= (or slope) represents the variable cost rate or the rate at which the independent
variable varies (choice-letter <a= is correct).
Choice-letter <b= is incorrect because the rate at which the dependent variable (Y) vary
depends on the value <x=, the independent variable. Choice letter <c= is incorrect because fixed costs
are not the same as the slope or variable cost rate. Choice-letter <d= is incorrect because the level of
the total variable costs (e.g., <bx=) is the total of slope (e.g., <b=) multiplied by the independent variable
(e.g., <x=).

Ivonne Corporation operates an automobile service facility that specializes in replacing mufflers on compact cars. The
following table shows the costs incurred during the month when 1,000 mufflers were replaced.

Number of muffler replacements 800 1,000 1,200


Total costs:
Fixed costs A 180,00 18,000 c 18,000
Variable costs b 9,600 12,000 D 216,000
Total costs e 2,7600 30,000 f 234,000

Cost of per muffler replacement Total Fixed Cost


Fixed costs g 22.5 h 18 i 15 Total # of muffler replacements
VC/muffler= Total VC
Variable costs j 12 k 12 l 12
Total # of muffler replacements
Total cost per muffler replacement m 34.5 n 30 o 27

Required: Fill in the missing amounts, labeled (a) through (o) in the table.

Naruto Corporation has observed the following processing costs at various levels over the last 15 periods:

Period Units produced Total cost

1 2,250 19,000

2 5,500 26,000

3 6,000 28,000

4 2,750 20,000

5 4,500 23,500

Downloaded by Sky Park (parksky512@gmail.com)


lOMoARcPSD|32626796

6 5,250 26,000

7 3,750 22,000

8 2,500 20,500

9 5,750 26,000

10 3,000 21,500

11 4,250 24,000

12 5,000 25,000

13 3,250 22,000

14 4,750 24,000

15 4,000 23,000

Required:
a) Using the high-low method, determine the variable cost per unit and total fixed costs.
b) Using the least-squared regression analysis, determine the variable cost per unit and total fixed costs.

A.

Units produced Cost


Highest (3) 6,000 28,000
Lowest (1) 2,250 19,000
Difference 3,750 9,000

VCR =  in Costs /  in Units = 9,000 / 3,750 = P2.4 variable cost/unit

Fixed Cost= Total Cost- Variable Cost


a= y - b(x)
a= 28000 - 2.4(6,000)
a= 28,000 - 14,400
a= 13,600

Downloaded by Sky Park (parksky512@gmail.com)


lOMoARcPSD|32626796

1. When inventories increase from one period to the next and all other factors remain constant, income under
direct costing:
A. will be irrelevant for decision making
B. will be smaller than under absorption costing
C. cannot be accurately computed
D. leads to smaller federal income tax payments
E. will be greater than under absorption costing

2. A basic tenet of direct costing is that period costs should be currently expensed. The rationale behind this
procedure is that:
A. allocation of period costs is arbitrary at best and could lead to erroneous decisions by management
B. since period costs will occur whether or not production occurs, it is improper to allocate these costs to
production and defer a current cost of doing business
C. period costs are uncontrollable and should not be charged to a specific product
D. period costs are generally immaterial in amount and the cost of assigning the amounts to specific products
would outweigh the benefits E. all of the above

3. All of the following statements related to the use of break-even analysis are true except:
A. a change in fixed costs changes the break-even point but not the contribution margin figure
B. a combined change in fixed and variable costs in the same direction causes a sharp change in the
breakeven point
C. a change in fixed costs changes the contribution margin figure but not the break-even point
D. a change in per-unit variable costs changes the contribution margin ratio E. a change in sales price changes
the break-even point

4. A major assumption concerning cost and revenue behavior that is important to the development of
breakeven charts is that:
A. all costs are variable
B. total costs are quadratic
C. costs and revenues are linear
D. the relevant range is greater than sales volume
E. costs will not exceed revenues
Y= a + bx

5. When referring to the "margin of safety," an accountant would be thinking of: EXCESS OF SALAES OVER
BREAKEVEN FIGURES
A. the excess of sales revenue over variable costs = CONTRIBUTION MARGIN
B. the excess of budgeted or actual sales over the contribution margin
C. the excess of budgeted or actual sales revenue over fixed costs
D. the excess of actual sales over budgeted sales
E. none of the above

6. Based on the cost-volume-profit chart in Figure 20-1 for a manufacturing company, the correct statement is:
A. line b graphs total fixed costs
B. point c represents the point at which the marginal contribution per unit increases
C. line d graphs total costs
D. area e (between lines b and d) represents the contribution margin E. area a represents the area of net loss

Downloaded by Sky Park (parksky512@gmail.com)


lOMoARcPSD|32626796

7. A valid assumption for cost-volume-profit analysis is:


A. an increase in fixed costs will cause the break-even point to rise (DECREASE!)
B. demand is constant regardless of price
C. a decrease in variable cost per unit will lower the break-even point (INCREASE CM BUT NOT BEP)
D. variable costs per unit are assumed to remain constant within the range of activity analyzed (WITHIN THE
RELEVANT RANGE)
E. E. all of the above are invalid assumptions

8. Total unit costs are


A. Relevant for cost-volume-profit analysis (CM is relevant)
B. Independent of the cost system used to generate them
C. Irrelevant in marginal analysis
D. Needed for determining product contribution

9. One of the major assumptions limiting the reliability of breakeven analysis is that
A. Efficiency and productivity will continually increase
B. Total variable costs will remain unchanged over the relevant range
C. Total fixed costs will remain unchanged over the relevant range
D. The cost of production factors varies with changes in technology

Limiting factors for Breakeven analysis

10. The most likely strategy to reduce the breakeven point would be to
A. Increase both the fixed cost and the contribution margin
B. Decrease both the fixed costs and the contribution margin
C. Decrease the fixed costs and increase the contribution margin
D. Increase the fixed costs and decrease the contribution margin

Downloaded by Sky Park (parksky512@gmail.com)


lOMoARcPSD|32626796

The following information pertains to Izzy Co.:

Sales (50,000 units)................................................................................................$1,000,000 Direct


materials and direct labor ................................................................................ 300,000
Factory overhead:
Variable......................................................................................................................... 40,000 Fixed
............................................................................................................... ............. 70,000
Selling and general expenses:
Variable......................................................................................................................... 10,000
Fixed ............................................................................................................................. 60,000
11. How much was Izzy's break-even point in number of units?
A. 18,571
B. 26,000
C. 9,848
D. 10,000
E. none of the above

12. What was Izzy's contribution margin ratio?


A. 65%
B. 59%
C. 35%
D. 66%
E. none of the above

13. Clark Co.'s operating percentages were as follows:

Sales 100%
Cost of sales:
Variable 50%
Fixed 10% 60%
Gross profit 40%
Other operating expenses
Variable 20%
Fixed 15% 35% Operating income 5%

Clark's sales totaled $2,000,000. At what sales level would Clark break even?
A. $1,900,000
B. $666,667
C. $1,250,000
D. $833,333
E. $1,666,667

Downloaded by Sky Park (parksky512@gmail.com)


lOMoARcPSD|32626796

14. A company has just completed the final development of its only product, general recombinant bacteria, that
kills most insects before dying. The product has taken three years and $6,000,000 to develop. The following
costs are expected to be incurred on a monthly basis for the production of 1,000,000 pounds of the new
product:

1,000,000 Pounds
Direct materials.................................................................................................... $ 300,000
Direct labor .......................................................................................................... 1,250,000
Variable overhead................................................................................................... 450,000
Fixed overhead................................................................................................ ... 2,000,000
Variable selling, general, and administrative expenses..................................... . 900,000
Fixed selling, general, and administrative expenses ........................................... 1,500,000
Total................................................................................................................... $ 6,400,000

At a sale price of $5.90 per pound, the sales in pounds necessary to ensure a $3,000,000 profit the first
year would be (to the nearest thousand pounds):
A. 13,017,000 pounds
B. 14,000,000 pounds
C. 15,000,000 pounds
D. 25,600,000 pounds
E. none of the above

15. Sanderson sells a single product for $50 that has a variable cost of $30. Fixed costs amount to $5 per unit
when anticipated sales targets are met. If the company sells one unit in excess of its break-even volume, the
bottom-line profit will be:
A. $15.
B. $20.
C. $50.
D. an amount that cannot be derived based on the information presented.
E. an amount other than those in choices "A," "B," and "C" but one that can be derived based on the
information presented.

Lamar & Co., makes and sells two types of shoes, Plain and Fancy. Data concerning these products are as
follows:

Plain Fancy
Unit selling price $20 $35
Variable cost per unit 12.00 24.50

Sixty percent of the unit sales are Plain, and annual fixed expenses
are $45,000.

16. The weighted-average unit contribution margin is: A.


$4.80.
B. $9.00.
C. $9.25.
D. $17.00.
E. an amount other than those above.

Downloaded by Sky Park (parksky512@gmail.com)


lOMoARcPSD|32626796

17. Assuming that the sales mix remains constant, the total number of units that the company must sell to break
even is: A. 2,432.
B. 2,647.
C. 4,737.
D. 5,000.
E. an amount other than those above.
18. Assuming that the sales mix remains constant, the number of units of Plain that the company must sell to
break even is: A. 2,000.
B. 3,000.
C. 3,375.
D. 5,000. E. 5,625.

19. Assuming that the sales mix remains constant, the number of units of Fancy that the company must sell to
break even is: A. 2,000.
B. 3,000.
C. 3,375.
D. 5,000. E. 5,625.

20. The following information relates to Day Company:

Sales revenue $12,000,000


Contribution margin 4,800,000
Net income 800,000

Day's operating leverage factor is: A.


0.067.
B. 0.167.
C. 0.400.
D. 2.500.
E. 6.000.

Downloaded by Sky Park (parksky512@gmail.com)


lOMoARcPSD|32626796

1. Consider the following three product costing alternatives: process costing, job order costing, and standard
costing. Which of these can be used in conjunction with absorption costing?
A. job order costing
B. standard costing
C. process costing
D. all of the above

2. If a firm produces more units than it sells, absorption costing, relative to variable costing, will result in
A. higher income and assets.
B. higher income but lower assets.
C. lower income but higher assets.
D. lower income and assets.

3. Under absorption costing, fixed manufacturing overhead could be found in all of the following except the
A. work-in-process account.
B. finished goods inventory account.
C. Cost of Goods Sold.
D. period costs.

4. What factor, related to manufacturing costs, causes the difference in net earnings computed using
absorption costing and net earnings computed using variable costing?
A. Absorption costing considers all costs in the determination of net earnings, whereas variable costing
considers fixed costs to be period costs.
B. Absorption costing allocates fixed overhead costs between cost of goods sold and inventories, and
variable costing considers all fixed costs to be period costs.
C. Absorption costing "inventories" all direct costs, but variable costing considers direct costs to be period
costs.
D. Absorption costing "inventories" all fixed costs for the period in ending finished goods inventory, but
variable costing expenses all fixed costs.

5. A basic tenet of variable costing is that period costs should be currently expensed. What is the rationale
behind this procedure?
A. Period costs are uncontrollable and should not be charged to a specific product.
B. Period costs are generally immaterial in amount and the cost of assigning the amounts to specific
products would outweigh the benefits.
C. Allocation of period costs is arbitrary at best and could lead to erroneous decision by management.
D. Because period costs will occur whether production occurs, it is improper to allocate these costs to
production and defer a current cost of doing business.

6. A firm presently has total sales of $100,000. If its sales rise, its
A. net income based on variable costing will go up more than its net income based on absorption costing
B. net income based on absorption costing will go up more than its net income based on variable costing.
C. fixed costs will also rise.
D. per unit variable costs will rise.

The following information is available for Ford Company for its first year of operations:

Sales in units 5,000

Production in units 8,000

Manufacturing costs:

Downloaded by Sky Park (parksky512@gmail.com)


lOMoARcPSD|32626796

Direct labor $3 per unit

Direct material 5 per unit

Variable overhead 1 per unit

Fixed overhead $100,000


Net income (absorption method) $30,000
Sales price per unit $40

7. Refer to Ford Company. If Ford Company had used variable costing, what amount of income before income
taxes would it have reported?
A. $30,000
B. ($7,500)
C. $67,500
D. can't be determined from the information given
Net Income--Absorption Costing $ 30,000
Fixed OH in Ending Inventory:
$100,000 * (3,000/8,000) ($37,500)
Net Loss--Variable Costing ($ 7,500)

8. Refer to Ford Company. What was the total amount of Selling,General and Administrative expense incurred
by Ford Company?
A. $30,000
B. $62,500
C. $6,000
D. can't be determined from the information given
Sales $200,000
COGS 107,500
Gross Profit 92,500
SG&A X
Net Income $ 30,000
X = $62,500

9. Refer to Ford Company. If Ford Company were using variable costing, what would it show as the value of
ending inventory?
A. $120,000
B. $64,500
C. $27,000
D. $24,000
3,000 units * $9.00/unit = $27,000

The following information has been extracted from the financial records of Clinton Corporation for its first year of
operations:

Units produced 10,000 Units sold 7,000


Variable cost per unit:
Direct material $8 Direct labor 9
Manufacturing overhead 3
SG&A 4
Fixed costs:
Manufacturing overhead $70,000

Downloaded by Sky Park (parksky512@gmail.com)


lOMoARcPSD|32626796

SG&A $30,000

10. Refer to Clinton Corporation. Based on absorption costing, Clinton Corporation's income in its first year of
operations will be
A. $21,000 higher than it would be under variable costing.
B. B. $70,000 higher than it would be under variable costing.
C. $30,000 higher than it would be under variable costing.
D. higher than it would be under variable costing, but the exact difference cannot be determined from the
information given.

3,000 unsold units * $7.00 fixed overhead/unit = $21,000 higher under absorption costing.

11. Refer to Clinton Corporation. Based on absorption costing, the Cost of Goods Manufactured for Clinton
Corporation's first year would be
A. $200,000. B. $270,000. C. $300,000. D. $210,000.

COGM = Variable Overhead + Fixed Overhead


COGM = (100,000 units * $20/unit) + $70,000
COGM = $270,000

12. Refer to Clinton Corporation. Based on absorption costing, what amount of period costs will Clinton
Corporation deduct?
A. $70,000 B. $79,000 C. $30,000 D.$58,000

Period costs = Variable SG&A + Fixed SG&A


$58,000 = (7,000 * $4) + $30,000

13. For its most recent fiscal year, a firm reported that its contribution margin was equal to 40 percent of sales
and that its net income amounted to 10 percent of sales. If its fixed costs for the year were $60,000, how
much were sales?
A. $150,000 B. $200,000 C. $600,000 D. can't be determined from the information given

Let S = Sales FC = .30S


Let CM = .40S $60,000 = .30S
Let NI = .10S S = $200,000

14. At its present level of operations, a small manufacturing firm has total variable costs equal to 75 percent of
sales and total fixed costs equal to 15 percent of sales. Based on variable costing, if sales change by $1.00,
income will change by
A. $0.25. B. $0.10. C.$0.75 D. can't be determined from the information given.

Let S = 1.00
Let VC = .75S
Let CM = .25S
Under variable costing every dollar of sales will increase net income by $0.25.

Downloaded by Sky Park (parksky512@gmail.com)


lOMoARcPSD|32626796

The following information regarding fixed production costs from a manufacturing firm is available for the current
year:

Fixed costs in the beginning inventory $16,000


Fixed costs incurred this period 100,000

15. Which of the following statements is not true?


A. The maximum amount of fixed production costs that this firm could deduct using absorption costs in the
current year is $116,000.
B. The maximum difference between this firm's the current year income based on absorption costing and
its income based on variable costing is $16,000.
C. Using variable costing, this firm will deduct no more than $16,000 for fixed production costs.
D. If this firm produced substantially more units than it sold in the current year, variable costing will
probably yield a lower income than absorption costing.

The following information was extracted from the first year absorption-based accounting records of Enigma
Corporation

Total fixed costs incurred $100,000


Total variable costs incurred 50,000
Total period costs incurred 70,000
Total variable period costs incurred 30,000
Units produced 20,000
Units sold 12,000
Unit sales price $12

16. Refer to Enigma Corporation. What is Cost of Goods Sold for Enigma Corporation's first year?
A. $80,000
B. $90,000
C. $48,000
D. can't be determined from the information given

Total variable manufacturing costs = $50,000 - 30,000 = $20,000


Total fixed period costs incurred = $70,000 - 30,000 = $40,000
Total fixed manufacturing costs = $100,000 - 40,000 = $60,000
Total manufacturing costs = $60,000 + $20,000 = $80,000
Percent of goods sold: 12,000/20,000 = 60%
$80,000 * 60% = $48,000

17. Refer to Enigma Corporation. If Enigma Corporation had used variable costing in its first year of operations,
how much income (loss) before income taxes would it have reported?
A. ($6,000)
B. $54,000
C. $26,000
D. $2,000
Sales $144,000
Less: Variable Costs
Manufacturing $20,000 * 60% 12,000
Period Costs $30,000 30.000
Contribution Margin $102,000
Fixed Costs 100,000
Variable Costing Net Income 2,000

Downloaded by Sky Park (parksky512@gmail.com)


lOMoARcPSD|32626796

18. Refer to Enigma Corporation. Based on variable costing, if Enigma had sold 12,001 units instead of 12,000,
its income before income taxes would have been
A. $9.50 higher.
B. $11.00 higher.
C. $8.50 higher.
D. $8.33 higher.

Sales Price per Unit: $12.00


Variable Costs per Unit ($50,000 / 20,000) 2.50
Contribution Margin $ 8.50

King Corporation produces a single product. The following cost structure applied to its first year of operations:

Variable costs:
SG&A $2 per unit
Production $4 per unit
Fixed costs (total incurred for the year):
SG&A $14,000
Production $20,000

19. Refer to King Corporation. Assume for this question only that during the current year King Corporation
manufactured 5,000 units and sold 3,800. There was no beginning or ending work-in-process inventory.
How much larger or smaller would King Corporation's income be if it uses absorption rather than variable
costing?
A. The absorption costing income would be $6,000 larger.
B. The absorption costing income would be $6,000 smaller.
C. The absorption costing income would be $4,800 larger.
D. The absorption costing income would be $4,000 smaller.

Add back fixed manufacturing portion of units unsold (1,200/5,000) * $20,000 = $4,800.

20. Refer to King Corporation. Assume for this question only that King Corporation manufactured and sold 5,000
units in the current year. At this level of activity it had an income of $30,000 using variable costing. What
was the sales price per unit?
A. $16.00 B. $18.80 C. $12.80 D. $14.80

Sales--5,000 units * $18.80/unit $94,000


Variable Costs:
Manufacturing 20,000
SG&A 10,000
Contribution Margin $64,000
Fixed Costs
Manufacturing 14,000
SG&A 20,000
Net Income $30,000

21. Refer to King Corporation. Assume for this question only that King Corporation produced 5,000 units and
sold 4,500 units in the current year. If King uses absorption costing, it would deduct period costs of
A. $24,000. B. $34,000. C. $27,000. D. $23,000.

Downloaded by Sky Park (parksky512@gmail.com)


lOMoARcPSD|32626796

Variable SG&A Costs (4,500 units * $2/unit) $ 9,000


Fixed SG&A Costs 14,000
Total period costs to be deducted $23,000

22. Refer to King Corporation. Assume for this question only that King Corporation manufactured 5,000 units
and sold 4,000 in the current year. If King employs a costing system based on variable costs, the company
would end the current year with a finished goods inventory of
A. $4,000. B. $8,000. C. $6,000. D. $5,000.

1,000 units * $4.00 variable cost per unit = $4,000

Three new companies (R, S, and T) began operations on January 1 of the current year. Consider the following
operating costs that were incurred by these companies during the complete calendar year:

Company R Company S Company T

Production in units 10,000 10,000 10,000


Sales price per unit $10 $10 $10
Fixed production costs $10,000 $20,000 $30,000
Variable production costs $30,000 $20,000 $10,000
Variable SG&A $10,000 $20,000 $30,000
Fixed SG&A $30,000 $20,000 $10,000

23. Refer to Companies R, S, and T. Based on sales of 7,000 units, which company will report the greater
income before income taxes if absorption costing is used?
A. Company R
B. Company S
C. Company T
D. All of the companies will report the same income.

Under absorption costing, the net income for all three companies is the same.

24. Refer to Companies R, S, and T. Based on sales of 7,000 units, which company will report the greater
income before income taxes if variable costing is used?
A. Company R
B. Company S
C. Company T
D. All of the companies will report the same income.

Since Company R has the largest variable manufacturing costs, income will increase by the
amount that was held in finished goods inventory.

25. Refer to Companies R, S, and T. Based on sales of 10,000 units, which company will report the greater
income before income taxes if variable costing is used?
A. Company R
B. Company S
C. Company T
D. All of the companies will report the same income before income taxes.

Since all the companies have the same net income and all had the same amount of sales,
all three companies would have the same net income under variable costing.

Downloaded by Sky Park (parksky512@gmail.com)


lOMoARcPSD|32626796

1. Additional output that results in a positive difference between differential revenues and differential
costs is beneficial to a company if and only if:
A. other sales are affected
B. other sales are unaffected and other unit costs are unaffected
C. other unit costs are increased and idle capacity is decreased
D. other sales are unaffected but other unit costs are increased
E. there is no idle capacity

2. The costing method used to determine the lowest price that could be quoted for a special order that
would use idle capacity within a production area is:
A. process
B. direct
C. standard
D. absorption
E. job order

3. The term "differential cost" refers to:


A. the profit foregone by selecting one alternative instead of another
B. a cost that does not entail any dollar outlay but that is relevant to the decision-making process
C. a cost that continues to be incurred even though there is no activity
D. a cost common to all alternatives in question and not clearly or practically allocable to anyof
the alternatives
E. the difference in total costs that results from selecting one alternative instead of another

4. For the past 12 years, the Jolt Company has produced the small electric motors that fit into its main
product line of dental drilling equipment. As materials costs have steadily increased, the controller
of the Jolt Company is reviewing the decision to continue to make the small motors and has
identified the following facts:

1. The equipment used to manufacture the electric motors has a book value of $150,000.
2. The space now occupied by the Electric Motor Manufacturing Department could be used
to eliminate the need for storage space now being rented.
3. Comparable units can be purchased from an outside supplier for $59.75.
4. Four of the people who work in the Electric Motor Manufacturing Department would be
terminated and given eight weeks of severance pay.
5. A $10,000 unsecured note is still outstanding on the equipment used in the manufacturing
process.

Which of the items above are relevant to the decision that the controller has to make?
A. 1, 2, 4, and 5
B. 1, 3, 4, and 5
C. 1, 3, and 4
D. 2, 3, and 4
E. 2, 3, 4, and 5

5. The minimum selling price that should be acceptable in a special order situation is equal to total
A. production cost.
B. variable production cost.
C. variable costs.
D. production cost plus a normal profit margin.

Downloaded by Sky Park (parksky512@gmail.com)


lOMoARcPSD|32626796

6. Which of the following costs is irrelevant in making a decision about a special order price if some
of the company facilities are currently idle?
A. direct labor
B. equipment depreciation
C. variable cost of utilities
D. opportunity cost of production

7. Assume a company produces three products: A, B, and C. It can only sell up to 3,000 units of each
product. Production capacity is unlimited. The company should produce the product (or products)
that has (have) the highest
A. contribution margin per hour of machine time.
B. gross margin per unit.
C. contribution margin per unit.
D. sales price per unit.

8. Fixed costs are ignored in allocating scarce resources because


A. they are sunk.
B. they are unaffected by the allocation of scarce resources.
C. there are no fixed costs associated with scarce resources.
D. fixed costs only apply to long-run decisions.

9. In a make or buy decision, the opportunity cost of capacity could


A. be considered to decrease the price of units purchased from suppliers.
B. be considered to decrease the cost of units manufactured by the company.
C. be considered to increase the price of units purchased from suppliers.
D. not be considered since opportunity costs are not part of the accounting records.

10. The opportunity cost of making a component part in a factory with excess capacity for which there
is no alternative use is
A. the total manufacturing cost of the component.
B. the total variable cost of the component.
C. the fixed manufacturing cost of the component.
D. zero.

11. Emerald Corporation has been manufacturing 5,000 units of Part 10541, which is used in the
manufacture of one of its products. At this level of production, the cost per unit of manufacturing
Part 10541 is as follows:

Direct material $2
Direct labor 8
Variable overhead 4
Fixed overhead applied 6
Total 20

Hamilton Company has offered to sell Emerald 5,000 units of Part 10541 for $19 a unit. Emerald
has determined that it could use the facilities currently used to manufacture Part 10541 to
manufacture Part RAC and generate an operating profit of $4,000. Emerald has also determined
that two-thirds of the fixed overhead applied will continue even if Part 10541 is purchased from
Hamilton. To determine whether to accept Hamilton’s offer, the net relevant costs to make are
A. $70,000.
B. $84,000.
C. $90,000.
D. $95,000.

Downloaded by Sky Park (parksky512@gmail.com)


lOMoARcPSD|32626796

12. Ely Electronics has the following standard costs and other data:

Part A4 Part B5
Direct materials $ .40 $ 8.00
Direct labor 1.00 4.70
Factory overhead 4.00 2.00
Unit standard cost $ 5.40 $ 14.70
Units needed per year 6,000 8,000
Machine hours per unit 4 2
Unit cost if purchased $ 5.00 $ 15.00

In past years, Ely has manufactured all of its required components; however, this year only
30,000 hours of otherwise idle machine time can be devoted to the production of components.
Accordingly, some of the parts must be purchased from outside suppliers. In producing parts,
factory overhead is applied at $1.00 per standard machine hour. Fixed capacity costs that will not
be affected by any make-or-buy decision represent 60% of the applied overhead. The 30,000
hours of available machine time are to be scheduled so that Ely realizes maximum potential cost
savings.

The relevant unit production costs that should be considered in the decision to schedule machine
time are:
A. $5.40 for A4 and $14.70 for B5
B. $5.00 for A4 and $15.00 for B5
C. $1.40 for A4 and $12.70 for B5
D. $3.00 for A4 and $13.50 for B5
E. none of the above

13. The Reno Company manufactures Part No. 498 for use in its production cycle. The cost per unit
for 20,000 units of Part No. 498 are as follows:

Direct materials $6
Direct labor 30
Variable overhead 12
Fixed overhead applied 16
64

The Tray Company has offered to sell 20,000 units of Part No. 498 to Reno for $60 per unit. Reno
will make the decision to buy the part from Tray if there is a savings of $25,000 for Reno. If Reno
accepts Tray's offer, $9 per unit of the fixed overhead applied would be totally eliminated.
Furthermore, Reno has determined that the released facilities could be used to save relevant costs
in the manufacture of Part No. 575. In order to have a savings of $25,000, the amount of relevant
costs that would be saved by using the released facilities in the manufacture of Part No. 575 would
have to be:
A. $80,000
B. $60,000
C. $125,000
D. $140,000
E. $85,000

Downloaded by Sky Park (parksky512@gmail.com)


lOMoARcPSD|32626796

14. At December 31, Zar Co. had a machine with an original cost of $84,000, accumulated depreciation
of $60,000, and an estimated salvage value of zero. On December 31, Zar was considering the
purchase of a new machine having a five-year life, costing $120,000, and having an estimated
salvage value of $20,000 at the end of the five years. In its decision concerning the possible
purchase of the new machine, how much should Zar consider to be a sunk cost at December 31?
A. $120,000
B. $100,000
C. $24,000
D. $4,000
E. none of the above

15. Doyle Company has 3 divisions: R, S, and T. Division R's income statement shows the following
for the year ended December 31:

Sales $1,000,000
Cost of goods sold (800,000)
Gross profit $ 200,000
Selling expenses (100,000)
Administrative expenses (250,000)
Net loss $ (150,000)

Cost of goods sold is 75 percent variable and 25 percent fixed. Of the fixed costs, 60 percent are
avoidable if the division is closed. All of the selling expenses relate to the division and would be
eliminated if Division R were eliminated. Of the administrative expenses, 90 percent are applied
from corporate costs. If Division R were eliminated, Doyle’s income would
A. increase by $150,000.
B. decrease by $ 75,000.
C. decrease by $155,000.
D. decrease by $215,000.

The Chip Division of Computer Solutions, Inc. produces a high-quality computer chip. Unit
production costs (based on capacity production of 100,000 units per year) follow:

Direct material $50


Direct labor 20
Overhead (20% variable) 10
Other information:
Sales price 100
SG&A costs (40% variable) 15

16. Refer to Chip Division of Computer Solutions, Inc. Assume, for this question only, that the Chip
Division is producing and selling at capacity. What is the minimum selling price that the division
would consider on a "special order" of 1,000 chips on which no variable period costs would be
incurred?
A. $100
B. $72
C. $81
D. $94

Downloaded by Sky Park (parksky512@gmail.com)


lOMoARcPSD|32626796

17. Refer to Chip Division of Computer Solutions, Inc. Assume, for this question only, that the Chip
Division is operating at a level of 70,000 chips per year. What is the minimum price that the division
would consider on a "special order" of 1,000 chips to be distributed through normal channels?
A. $78
B. $95
C. $100
D. $81

18. Refer to Chip Division of Computer Solutions, Inc. Assume, for this question only, that the Chip
Division is presently operating at a level of 80,000 chips per year. Accepting a "special order" on
2,000 chips at $88 will
A. increase total corporate profits by $4,000.
B. increase total corporate profits by $20,000.
C. decrease total corporate profits by $14,000.
D. decrease total corporate profits by $24,000.

19. Stewart Industries has been producing two bearings, components B12 and B18, for use in
production. Data regarding these two components are:

B12 B18
Machine hours required per unit 2.5 3.0
Standard cost per unit
Direct material $ 2.25 $ 3.75
Direct labor 4.00 4.50
Manufacturing overhead
Variable1 2.00 2.25
Fixed2 3.75 4.50
$ 12.00 $15.00
1
Variable manufacturing overhead is applied on the basis of direct labor hours.
2
Fixed manufacturing overhead is applied on the basis of machine hours.

Stewart's annual requirement for these components is 8,000 units of B12 and 11,000 units of B18.
Recently, Stewart's management decided to devote additional machine time to other product lines
resulting in only 41,000 machine hours per year that can be dedicated to the production of the
bearings. An outside company has offered to sell Stewart the annual supply of the bearings at
prices of $11.25 for B12 and $13.50 for B18. Stewart wants to schedule the otherwise idle 41,000
machine hours to produce bearings so that the company can minimize its costs (maximize its net
benefits).

The net benefit (loss) per machine hour that would result if Stewart Industries accepts the supplier's
offer of $13.50 per unit for component B18 is:
A. $.50
B. $(1.00)
C. $1.50
D. $(1.75)
E. some amount other than those given above

Downloaded by Sky Park (parksky512@gmail.com)


lOMoARcPSD|32626796

20. A company owns equipment that is used to manufacture important parts for its production process.
The company plans to sell the equipment for $10,000 and to select one of the following two
alternatives: (1) acquire new equipment for $80,000, or (2) purchase the important parts from an
outside company at $4 per part. To select the best alternative, the company should compare the
cost of manufacturing the parts:
A. plus $80,000 to the cost of buying the parts less $10,000
B. to the cost of buying the parts less $10,000
C. less $10,000 to the cost of buying the parts
D. plus $80,000 to the cost of buying the parts
E. none of the above

21. The objective in solving the linear programming problem is to determine the optimal levels of the
A. coefficients.
B. dependent variables.
C. independent variables.
D. slack variables.

22. The feasible region for an LP solution is


A. defined only by binding constraints on the optimal solution.
B. defined as the solution space that satisfies all constraints.
C. identified by iso-cost and iso-profit lines.
D. identified by all of the above.

23. Which of the following variables is associated with the "less than or equal to" constraints?

Surplus Slack
A. yes yes
B. yes no
C. no yes
D. no no

24. The graphical approach to solving a linear programming problem becomes much more complex
when

there are more than two constraints decision variables


A. yes no
B. no yes
C. yes yes
D. no no

25. A linear programming solution


A. always involves more than one constraint.
B. always involves a corner point.
C. is the one with the highest vertex coordinates.
D. is provided by the input-output coefficients.

Downloaded by Sky Park (parksky512@gmail.com)


lOMoARcPSD|32626796

In the two following constraint equations, X and Y represent two products (in units) produced by
the Uncommon Products Corporation.

Constraint 1: 3X + 5Y < 4,200


Constraint 2: 5X + 2Y > 3,000

26. Refer to Uncommon Products Corporation. What is the maximum number of units of Product X that
can be produced?
A. 4,200
B. 3,000
C. 600
D. 1,400

27. Refer to Uncommon Products Corporation. What is the feasible range for the production of Y?
A. 840 to 1,500 units
B. 0 to 840 units
C. 0 to 631 units
D. 0 to 1500 units

28. Refer to Uncommon Products Corporation. A solution of X = 500 and Y = 600 would violate
A. Constraint 1.
B. Constraint 2.
C. both constraints.
D. neither constraint.

29. One constraint in an LP problem is: 12X + 7Y > 4,000. If the optimal solution is X = 100 and
Y = 500, this resource has
A. slack variable of 700.
B. surplus variable of 700.
C. output coefficient of 700.
D. none of the above.

30. Consider the following linear programming problem and assume that non-negativity constraints
apply to the independent variables:

Max CM = $14X + $23Y


Subject to
Constraint 1: 4X + 5Y < 3,200
Constraint 2: 2X + 6Y < 2,400

Which of the following are feasible solutions to the linear programming problem?
A. X = 600, Y = 240
B. X = 800, Y = 640
C. X = 0, Y = 400
D. X = 1,200, Y = 0

Downloaded by Sky Park (parksky512@gmail.com)


lOMoARcPSD|32626796

1. Which of the following objectives is not a primary purpose of preparing a budget?


a. To provide a basis for comparison of actual performance
b. To communicate the company’s plans throughout the entire business organization
c. To control income and expenditure in a given period.
d. To make sure the company expands its operations.

2. Budget slack is a condition in which


a. demand is low at various times of the year.
b. excess machine capacity exists in some areas of the plant.
c. there is an intentional overestimate of expenses or an underestimate of revenues.
d. managers grant favored employees extra time off.

3. Ineffective budgets and/or control systems are characterized by the use of


a. budgets as a planning tool only and disregarding them for control purposes.
b. budgets for motivation.
c. budgets for coordination.
d. the budget for communication.

4. For better management acceptance, the flow of data to be used for budgeting should begin with
a. Accounting department
b. Lower levels of management
c. Top management
d. Budget committee

participatory

5. These statements are proper to the budgeting process except:


a. It is a part of management’s responsibility to plan the use of its resources.
b. It is a tool to orchestrate the various functions of operations in a business.
c. The involvement of the various levels of individuals in the company is necessary to gain its
acceptance and attain its goals.
d. Actual results need not be compared with plan, since the process ends after budget is
approved.

6. Just-in-time manufacturers are more likely than conventional manufacturers to


a. Prepare production budgets without a sales forecast.
b. Budget materials purchases equal to the current month’s needs for production.
c. Budget unit production for the month at greater than budgeted unit sales for the month. d.
Experience cash shortages.

7. On budgeting, all of the following are not valid, except


a. Responsibility budget identifies revenue and costs with the individual responsible for their
incurrence.
b. The best way to establish budget figures is to use last year’s actual cost and activity data as
this year’s budget estimates.
c. A sales budget and a sales forecast are the same thing.
d. The primary purpose of the cash budget is to show the expected cash balance at the end of
the budget period.

8. Which of the following statements is True?


a. Under zero-based budgeting, a manager is required to start at zero budget levels each
period, as if the programs involved were being initiated for the first time.

Downloaded by Sky Park (parksky512@gmail.com)


lOMoARcPSD|32626796

b. The primary purpose of the cash budget is to show the expected cash balance at the end of
the budget period.
c. Budget data are generally prepared by top management and distributed downward in an
organization.
d. The budget committee is responsible for preparing detailed budget figures in an organization.

9. This budgeting system places the burden of proof on the manager to justify authority to spend any
money whether or not there was spending in the previous period. Different ways of performing the
same activity and different levels of effort for the activity is evaluated. This system is called a.
Scenario budgeting.
b. Budgeting by alternatives.
c. Zero-based budgeting.
d. Budgeting by responsibility and authority.

10. In zero-based budgeting, which of the following statements are True?


1. All activities in the company are organized into break-up units called packages.
2. All costs have to be justified every budgeting period.
3. The process is not time consuming since justification of costs can be done as a routine matter.

a. All three statements.


b. Statement 1 only.
c. Statements 1 and 2 only.
d. Statement 2 and 3 only.

11. Which of these statements are advantages of profit planning?


1. Develops profit-mindedness, encourages cost consciousness and resources utilization
throughout the company. ✓
2. Provides vehicle to communicate objectives, gain support for the plan, of what is expected,
thereby developing a sense of commitment to achieve established goals. ✓
3. Provides yardstick to evaluate actual performance; encouraging efficiency, increasing output and
reducing cost. ✓
4. Provides a sense of direction for the company and enhances coordination of business activity. ✓
5. Eliminates or takes over the role of administration by providing detailed information that allows
executives to operate toward achievement of the organization’s objectives. ×

a. Statements 3, 4, and 5 only.


b. Statements 1, 3, and 4 only.
c. All five statements.
d. Statements 1, 2, 3, and 4 only.

12. For a company that does not have resource limitations in what sequence would the budgets be
prepared?
1. cash budget
2. sales budget
3. inventory budgets
4. production budgets
5. purchase budgets

a. sequence 2, 3, 4, 5 and 1
b. sequence 2, 4, 3, 5 and 1
c. sequence 2, 3, 4,1 and 5
d. sequence 4, 3, 2, 1 and 5

Downloaded by Sky Park (parksky512@gmail.com)


lOMoARcPSD|32626796

13. If a company has a policy of maintaining an inventory of finished goods at a specified percentage of
the next month's budgeted sales, budgeted production for January will exceed budgeted sales for
January when budgeted
a. February sales exceed budgeted January sales.
b. January sales exceed budgeted December sales.
c. January sales exceed budgeted February sales.
d. December sales exceed budgeted January sales.

14. Net cash inflow is given too much emphasis by managers today, for they know that cash is the
common cause of business failures. Net cash inflow is equal to
a. Cash balance at the beginning + cash receipts – cash disbursements
b. Cash balance at the end of last month + cash from all sources of revenue – revenue payments
c. Cash received during the period minus cash disbursements during the period
d. Cash sales and collections of accounts receivable minus revenue and capital expenditures

15. If cash receipts from customers are greater than sales, which of the following is most likely to be true?
a. Accounts receivable will decrease.
b. Cash balance will increase.
c. Outstanding debt will decrease.
d. The company will show a profit.

16. Ebony Company has the following expected pattern of collections on credit sales: 70 percent
collected in the month of sale, 15 percent in the month after the month of sale, and 14 percent in the
second month after the month of sale. The remaining 1 percent is never collected. At the end of May,
Ebony Company has the following accounts receivable balances:

From April sales $21,000


From May sales 48,000

Ebony's expected sales for June are $150,000. What were total sales for April? a.
$150,000
b. $72,414
c. $70,000
d. $140,000
Balance %
AR - April sales 21,000 15% 140,000 Sales - April
AR - May sales 48,000
Expected sales - June 150,000

Checking
Sale: 140,000
Collected same month 70% - 98,000
AR Beg, May 42,000
Collected month after sale 15% - 21,000
AR End, May 21,000
Remaining uncollected 15% 21,000 14% collectible in June + 1% uncollectible
-

Downloaded by Sky Park (parksky512@gmail.com)


lOMoARcPSD|32626796

Triple P Companies
Cash Budget

Company A Company B Company C


Beginning cash balance $100 $300 $700
Cash collections ? 400 ?
Cash disbursements 500 ? 600
Cash excess (shortage) ? ? 400
Borrowing (repayments) 300 100 ?
Ending cash 200 200 100

Cash Budget
Company
Company B Company C
A
Beginning cash balance 100 300 700
Cash collections 300 400 300
Cash disbursements 500 600 600
Cash excess (shortage) -100 100 400
Borrowing (repayments) 300 100 300
Ending cash 200 200 100

17. Refer to Triple P Companies. For Company A,


what are the budgeted cash collections? Ending cash 200
a. $700 Borrowing (repayments) (300)
b. $500 Cash excess (shortage) (100)
c. $300 Cash disbursements 500
d. $400 Beginning cash balance (100)
Cash collections 300

18. Refer to Triple P Companies. For Company B, Ending cash 200


what are the budgeted cash disbursements? Borrowing (repayments) (100)
a. $600 Cash excess (shortage) 100
b. $700 Cash collections (400)
c. $500 Beginning cash balance (300)
d. $400
Cash collections (600)

19. Refer to Triple P Companies. For Company C,


what are the budgeted cash collections? Cash excess (shortage) 400
a. $200 Cash disbursements 600
b. $300 Beginning cash balance (700)
c. $400 Cash collections 300
d. $500

Downloaded by Sky Park (parksky512@gmail.com)


lOMoARcPSD|32626796

Krebs Company is preparing its Manufacturing Overhead budget for the second quarter of the year.
Budgeted variable factory overhead is $3.00 per unit produced; budgeted fixed factory overhead is
$75,000 per month, with $16,000 of this amount being factory depreciation.

20. Refer to Krebs Company. If the budgeted Budgeted VOH 3.00


production for April is 6,000 units, then
the total budgeted factory overhead for Budgeted FOH 75,000
April is:
Budgeted FOH - Depreciation 16,000
a. $77,000
Budgeted FOH - Others 59,000
b. $82,000
c. $85,000
d. $93,000 Budgeted production 6,000 5,000

21. Refer to Krebs Company. If the budgeted Budgeted VOH 18,000 15,000
production for May is 5,000 units, then Budgeted FOH 75,000 75,000
the total budgeted factory overhead per Total Budgeted OH 93,000 90,000
unit: Per unit 18.00
a. $15
b. $18 Budgeted cash disbursement 80,000
c. $20 Budgeted FOH - Depreciation 16,000
d. $22 Budgeted OH 96,000
Budgeted FOH 75,000
22. 74. Refer to Krebs Company. If the Budgeted VOH 21,000
budgeted cash disbursements for factory Per unit 3.00
overhead for June are $80,000, then the Budgeted production 7,000
budgeted production for June must be:
a. 7,400 units
b. 6,200 units
c. 6,500 units
d. 7,000 units

Downloaded by Sky Park (parksky512@gmail.com)


lOMoARcPSD|32626796

Evita Company, a reseller of women’s fashions, has budgeted its activity for March. The budget
information is presented below:
I. Sales are $550,000. All sales are cash.
II. Merchandise inventory on February 28 is $300,000 III. Budgeted
depreciation for March is $35,000.
IV. Cash in bank on March 1 is $25,000.
V.Selling and administrative expenses are budgeted at $60,000 for March and are paid in cash. VI. VI.
The planned merchandise inventory on March 31 is $270,000.
VII. VII. The invoice cost for merchandise purchases represents 75% of sales price. All purchases are
paid for in cash.
Cash sales 550,000
23. Refer to Evita Company. The budgeted
cash receipts for March are: Inventory, beg 300,000
a. $412,500 Budgeted depreciation 35,000
b. $585,000 Cash in bank, beg 25,000
c. $137,500 Selling and Admin 60,000
d. $550,000 Inventory, end - plannned 270,000
Purchases 75% 412,500
24. Refer to Evita Company. The budgeted
cash disbursements for March are: Planned cash receipts 550,000
a. $382,500  All cash
b. $472,500 sales
c. $442,500 Inventory, end - plannned 270,000
d. $477,500 COGS 412,500
Inventory, beg (300,000 )
25. Refer to Evita Company. The budgeted Planned purchases 382,500
net income for December is: Selling and Admin 60,000
a. $107,500 Planned cash disbursements 442,500
b. $ 42,500
c. $137,500 Cash sales 550,000
d. $ 77,500 COGS (412,500)
Selling and Admin (60,000)
Depreciation (35,000)
42,500

26. For the month of October, P Corp. predicts total cash collections to be $1 million. Also for October, P
Corp. estimates that its beginning cash balance will be $50,000 and that it will borrow cash in the
amount of $70,000. If P Corp. estimates an ending cash balance of $30,000 for October, what must
its projected cash disbursements be?
a. $1,090,000
b. $1,120,000 Beginning cash balance 50,000
c. $1,070,000 Cash receipts 70,000
d. $1,020,000 Cash collections 1,000,000
Ending cash balance (30,000)
Projected cash disbursements 1,090,000

Downloaded by Sky Park (parksky512@gmail.com)


lOMoARcPSD|32626796

27. Harrison Company manufactures card tables. The company has a policy of maintaining a finished
goods inventory equal to 40 percent of the next month's planned sales. Each card table requires 3
hours of labor. The budgeted labor rate for the coming year is $13 per hour. Planned sales for the
months of April, May, and June are respectively 4,000; 5,000; and 3,000 units. What is Harrison
Company’s budgeted direct labor cost for May?
a. $54,600
b. $163,800 based kay Hennessy (makes more sense)
c. $226,200
d. $179,400 based sa Solution Manual ni Raiborn

April May June


Sales 4,000 5,000 3,000

Planned sales 5,000


Planned FG, end 40% 1,200
Planned FG, beg 40% (2,000)
4,200
DL Cost ($13 x 3 hours) 39
163,800

28. Budgeted sales for the first six months the year for Gibson Corporation are listed below:

January February March April May June

Units 6,000 7,000 8,000 7,000 5,000 4,000

Gibson Corporation has a policy of maintaining an inventory of finished goods equal to 40 percent of
the next month's budgeted sales. How many units has Gibson Corporation budgeted to produce in
the first quarter of the year?
a. 21,400 units
b. 20,600 units
c. 19,000 units
d. 23,000 units
40%
January February March April May June
Units 6,000 7,000 8,000 7,000 5,000 4,000
FG, end 2,800 3,200 2,800
FG, beg (2,400) (2,800) (3,200)
Q1 planned production 6,400 7,400 7,600

Total 21,400

Downloaded by Sky Park (parksky512@gmail.com)


lOMoARcPSD|32626796

29. Weaver Co. manufactures card tables. The company has a policy of maintaining a finished goods
inventory equal to 40 percent of the next month's planned sales. Each card table requires 3 hours of
labor. The budgeted labor rate for the coming year is $13 per hour. Planned sales for the months of
April, May, and June are respectively 4,000; 5,000; and 3,000 units. The budgeted direct labor cost
for June for Weaver Co. is $136,500. What are budgeted sales for July for Weaver Co.? a. 3,500
units
b. 4,250 units
c. 4,000 units
d. 3,750 units
April May June July
Sales 4,000 5,000 3,000

DL Cost 136,500
DL Cost ($13 x 3 hours) 39
Total production - June 3,500
Planned FG, beg 40% 1,200
4,700
June sales 3,000
Planned FG, end 1,700
FG % 40%
Budgeted sales 4,250

30. Production of Product X has been budgeted at 200,000 units for May. One unit of X requires 2 lbs. of
raw material. The projected beginning and ending materials inventory for May are:

Beginning inventory: 2,000 lbs.


Ending inventory: 10,000 lbs.

How many lbs. of material should be purchased during May?


a. 192,000
b. 208,000
c. 408,000
d. 416,000
Beginning inventory 2,000
Planned production (400,000)
Ending inventory (10,000)
Purchases (408,000)

Downloaded by Sky Park (parksky512@gmail.com)


lOMoARcPSD|32626796

1. Which of the following statements regarding standard cost systems is true?


a. Favorable variances are not necessarily good variances.
b. Managers will investigate all variances from standard.
c. The production supervisor is generally responsible for material price
variances.
d. Standard costs cannot be used for planning purposes since costs normally
change in the future.

2. In a standard cost system, Work in Process Inventory is ordinarily debited with


a. actual costs of material and labor and a predetermined overhead cost for
overhead.
b. standard costs based on the level of input activity (such as direct labor
hours worked).
c. standard costs based on production output.
d. actual costs of material, labor, and overhead.

3. A company wishing to isolate variances at the point closest to the point of responsibility will
determine its material price variance when
a. material is purchased.
b. material is issued to production.
c. material is used in production.
d. production is completed.

4. A company would most likely have an unfavorable labor rate variance and a favorable labor
efficiency variance if
a. the mix of workers used in the production process was more experienced
than the normal mix.
b. the mix of workers used in the production process was less experienced
than the normal mix.
c. workers from another part of the plant were used due to an extra heavy
production schedule.
d. the purchasing agent acquired very high quality material that resulted in
less spoilage.

5. If actual direct labor hours (DLHs) are less than standard direct labor hours allowed and overhead
is applied on a DLH basis, a(n)
a. favorable variable overhead spending variance exists.
b. favorable variable overhead efficiency variance exists.
c. favorable volume variance exists.
d. unfavorable volume variance exists.

6. In analyzing manufacturing overhead variances, the volume variance is the difference between the
a. amount shown in the flexible budget and the amount shown in the debit side of the overhead
control account.
b. predetermined overhead application rate and the flexible budget application rate times actual
hours worked.

Downloaded by Sky Park (parksky512@gmail.com)


lOMoARcPSD|32626796

c. budget allowance based on standard hours allowed for actual production for the period and the
amount budgeted to be applied during the period.
d. actual amount spent for overhead items during the period and the overhead amount applied to
production during the period.

7. The efficiency variance computed on a three-variance approach is


a. equal to the variable overhead efficiency variance computed on the four-
variance approach.
b. equal to the variable overhead spending variance plus the variable
overhead efficiency variance computed on the four-variance approach.
c. computed as the difference between applied variable overhead and actual
variable overhead.
d. computed as actual variable overhead minus the flexible budget for
variable overhead based on actual hours worked.

8. If all sub-variances are calculated for labor, which of the following cannot be determined? a. labor
rate variance
b. actual hours of labor used
c. reason for the labor variances
d. efficiency of the labor force

9. A company has a favorable variable overhead spending variance, an unfavorable variable


overhead efficiency variance, and underapplied variable overhead at the end of a period. The
journal entry to record these variances and close the variable overhead control account will show
which of the following?

VOH spending variance VOH efficiency variance VMOH


a. debit credit credit
b. credit debit credit
c. debit credit debit
d. credit debit debit

10. A company may set predetermined overhead rates based on normal, expected annual, or
theoretical capacity. At the end of a period, the fixed overhead spending variance would
a. be the same regardless of the capacity level selected.
b. be the largest if theoretical capacity had been selected.
c. be the smallest if theoretical capacity had been selected.
d. not occur if actual capacity were the same as the capacity level selected.

Marley Company

The following July information is for Marley Company:

Standards:
Material 3.0 feet per unit @ $4.20 per foot
Labor 2.5 hours per unit @ $7.50 per hour

Downloaded by Sky Park (parksky512@gmail.com)


lOMoARcPSD|32626796

Actual:
Production 2,750 units produced during the month
Material 8,700 feet used; 9,000 feet purchased @ $4.50 per foot
Labor 7,000 direct labor hours @ $7.90 per hour

(Round all answers to the nearest dollar.)

11. Refer to Marley Company. What is the material price variance (calculated at point of purchase)?
a. $2,700 U
b. $2,700 F
c. $2,610 F
d. $2,610 U

Material Price Variance = (Actual Price – Standard Price) * Actual Quantity Purchased
= ($4.50 - $4.20) * 9,000 feet purchased
= $2,700 U

12. Refer to Marley Company. What is the material quantity variance?


a. $3,105 F
b. $1,050 F
c. $3,105 U
d. $1,890 U

Material Quantity Variance = (Actual Quantity used – Standard Quantity allowed for input) *
Standard Price
= (8,700 - (2,750 * 3)) * $4.20
= $1,890 U

13. Refer to Marley Company. What is the labor rate variance?


a. $3,480 U
b. $3,480 F
c. $2,800 U
d. $2,800 F

Labor Rate Variance = (AP - SP) * AQ


= ($7.90 - $7.50) * 7,000 hr used
= $2,800 U

14. Refer to Marley Company. What is the labor efficiency variance?


a. $1,875 U
b. $938 U
c. $1,875 U
d. $1,125 U

Labor Efficiency Variance = (AQ - SQ) * SP


= (7,000 hr - (2.5 hr/unit * 2,750 units)) * $7.50

Downloaded by Sky Park (parksky512@gmail.com)


lOMoARcPSD|32626796

= $938 U (rounded)

Forrest Company

Forrest Company uses a standard cost system for its production process and applies overhead based
on direct labor hours. The following information is available for August when Forrest made 4,500 units:

Standard:
DLH per unit 2.50
Variable overhead per DLH $1.75
Fixed overhead per DLH $3.10
Budgeted variable overhead $21,875
Budgeted fixed overhead $38,750

Actual:
Direct labor hours 10,000
Variable overhead $26,250
Fixed overhead $38,000

15. Refer to Forrest Company. Using the one-variance approach, what is the total overhead variance?
a. $6,062.50 U
b. $3,625.00 U
c. $9,687.50 U
d. $6,562.50 U
e.
Total Variance = Actual Overhead - Applied Overhead
= $(26,250 + 38,000) - ($(1.75 + 3.10) * 2.50 hrs/unit * 4,500 units)
= $64,250.00 - $54,462.50
= $9,687.50U

16. Refer to Forrest Company. Using the two-variance approach, what is the controllable variance? a.
$5,812.50 U
b. $5,812.50 F
c. $4,375.00 U
d. $4,375.00 F

Controllable Variance = Actual Overhead - Budgeted Overhead Based on Standard Quantity


= $64,250.00 - $((4,500 units * 2.5 DLH/unit * $1.75) + 38,750)
= $(64,250 - $58,437.50)
= $5,812.50 U

17. Refer to Forrest Company. Using the two-variance approach, what is the noncontrollable variance?
a. $3,125.00 F
b. $3,875.00 U
c. $3,875.00 F
d. $6,062.50 U

Downloaded by Sky Park (parksky512@gmail.com)


lOMoARcPSD|32626796

Uncontrollable Variance = Budgeted Overhead Based on SQ - Applied Overhead


= $(58,437.50 - 54,562.50)
= $3,875.00 U

18. 68. Refer to Forrest Company. Using the three-variance approach, what is the spending variance?
a. $4,375 U
b. $3,625 F
c. $8,000 U
d. $15,750 U

OH Spending Variance = Actual OH - Budgeted OH based upon Inputs Used


= $64,250 - ((10,000 hrs * $1.75) + $38,750)
= $(64,250 - 56,250)
= $8,000.00 U

19. Refer to Forrest Company. Using the three-variance approach, what is the efficiency variance? a.
$9,937.50 F
b. $2,187.50 F
c. $2,187.50 U
d. $2,937.50 F

OH Efficiency Variance = Budgeted OH based on Actual - Budgeted OH based on Standard


= ((10,000 * $1.75)+ $38,750) - ((4,500 * 2.50 * $1.75) + $38,750)
= $(56,250.00 - 58,437.50)
= $2,187.50 F

20. Refer to Forrest Company. Using the three-variance approach, what is the volume variance? a.
$3,125.00 F
b. $3,875.00 F
c. $3,875.00 U
d. $6,062.50 U
Volume Variance = Budget Based on Standard Quantity - Overhead Applied
= $(58,437.50 - 54,562.00)
= $3,875.00 U

21. Refer to Forrest Company. Using the four-variance approach, what is the variable overhead
spending variance? a. $4,375.00 U
b. $4,375.00 F
c. $8,750.00 U
d. $6,562.50 U
Variable Overhead Spending Variance = Actual VOH - Budgeted VOH/Actual Quantity
= $26,250.00 - (10,000 * $1.75/VOH hr)
= $(26,250.00 - 17,500.00)
= $8,750.00 U

22. Refer to Forrest Company. Using the four-variance approach, what is the variable overhead
efficiency variance? a. $2,187.50 U

Downloaded by Sky Park (parksky512@gmail.com)


lOMoARcPSD|32626796

b. $9,937.50 F
c. $2,187.50 F
d. $2,937.50 F
VOH Efficiency Variance = Budgeted VOH based on Actual - Budgeted VOH/Standard Qty
= ((10,000 * $1.75/hr) - ((4,500 * 2.50hrs/unit * $1.75/hr))
= $(17,500.00 - 19,687.50)
= $2,187.50 F

23. Refer to Forrest Company. Using the four-variance approach, what is the fixed overhead spending
variance?
a. $7,000 U
b. $3,125 F
c. $750 U
d. $750 F
Fixed OH Spending Variance = Actual Fixed OH - Applied Fixed OH
= $(38,000 - 38,750)
= $750 F

24. Refer to Forrest Company. Using the four-variance approach, what is the volume variance? a.
$3,125 F
b. $3,875 F
c. $6,063 U
d. $3,875 U

Volume Variance = Budget Based on Standard Quantity - Overhead Applied


= $(58,437.50 - 54,562.00)
= $3,875.00 U

Ultra Shine Company

Ultra Shine Company manufactures a cleaning solvent. The company employs both skilled and unskilled
workers. To produce one 55-gallon drum of solvent requires Materials A and B as well as skilled labor
and unskilled labor. The standard and actual material and labor information is presented below:

Standard:
Material A: 30.25 gallons @ $1.25 per gallon
Material B: 24.75 gallons @ $2.00 per gallon
Skilled Labor: 4 hours @ $12 per hour
Unskilled Labor: 2 hours @ $ 7 per hour

Actual:
Material A: 10,716 gallons purchased and used @ $1.50 per gallon
Material B: 17,484 gallons purchased and used @ $1.90 per gallon
Skilled labor hours: 1,950 @ $11.90 per hour
Unskilled labor hours: 1,300 @ $7.15 per hour

During the current month Ultra Shine Company manufactured 500 55-gallon drums.

Downloaded by Sky Park (parksky512@gmail.com)


lOMoARcPSD|32626796

Round all answers to the nearest whole dollar.

25. Refer to Ultra Shine Company. What is the total material price variance?
a. $877 F
b. $877 U
c. $931 U
d. $931 F
Total Material Price Variance = Actual Mix,Qty,Price - Actual Mix,Quantity,Std Price
= $(49,294 - 48,363)
= $931 U

26. Refer to Ultra Shine Company. What is the total material mix variance?
a. $3,596 F
b. $3,596 U
c. $4,864 F
d. $4,864 U
Total Material Mix Variance = Actual Mix,Qty, Std Price - Std Mix, Price,Actual Qty
= $(48,363 - 44,767)
= $3,596 U

27. Refer to Ultra Shine Company. What is the total material yield variance?
a. $1,111 U
b. $1,111 F
c. $2,670 U
d. $2,670 F
Material Yield Variance = Std Mix, Std Price,Actual Qty - Std Mix, Qty, Price
= $(44,767 - $43,656)
= $1,111 U

28. Refer to Ultra Shine Company. What is the labor rate variance?
a. $0
b. $1,083 U
c. $2,583 U
d. $1,083 F
Labor Rate Variance = Actual Mix, Qty,Price - Actual Mix,Qty,Std Price
= $(32,500 - 32,500)
= $0

29. Refer to Ultra Shine Company. What is the labor mix variance?
a. $1,083 U
b. $2,588 U
c. $1,083 F
d. $2,588 F
Labor Mix Variance = Actual Mix,Qty, Std Price - Std Mix, Actual Qty, Std Price
= $(32,500 - 33,583)

Downloaded by Sky Park (parksky512@gmail.com)


lOMoARcPSD|32626796

= $1,083 F

30. Refer to Ultra Shine Company. What is the labor yield variance?
a. $2,583 U
b. $2,583 F
c. $1,138 F
d. $1,138 U

Labor Yield Variance = Std Mix, Act Qty, Std Price - Std Mix, Qty, Price
= $(33,583 - $31,000)
= $2,583 U

Downloaded by Sky Park (parksky512@gmail.com)


lOMoARcPSD|32626796

1. Internal reports prepared under the responsibility accounting approach should highlight:
A. cost properly allocable to the cost center under generally accepted accounting principles
B. fixed cost of production
C. variable cost of production
D. conversion cost
E. controllable cost

2. A company has three producing departments and one service department. Due to a scheduling
error in the service department, an unfavorable variance was created. A sound responsibility
accounting system would dictate that the variance be:
A. ignored
B. allocated to producing departments, but not on the same basis as ordinary charges for use
of the service
C. charged to the service department causing the variance and not allocated to other
departments
D. allocated to both producing and service departments
E. allocated to producing departments on the basis of usage

3. The cost item least likely to appear in a performance report based on responsibility accounting
techniques for the supervisor of an assembly line in a large manufacturing situation is:
A. materials
B. repairs and maintenance
C. direct labor
D. other indirect labor
E. supervisor's salary

4. Responsibility reports should possess all of the following characteristics except:


A. being issued with regularity
B. fitting the organization chart
C. being consistent in form and content each time they are issued
D. being stated only in dollars for operating management
E. comparing budgeted with actual figures

5. Controllable costs are:


A. costs that fluctuate in total in response to small changes in the rate of capacity utilization
B. costs that will be unaffected by current managerial decisions
C. costs that management decides to incur in the current period to enable the company to
achieve objectives other than filling customers' orders
D. costs that are likely to respond to the amount of attention devoted to them by a specified
manager
E. costs that are governed mainly by past decisions that established present levels of
operating and organizational capacity and that change slowly only in response to changes
in capacity

6. In a responsibility accounting system, costs are classified into categories on the basis of:
A. prime and overhead costs
B. administrative and nonadministrative costs
C. controllable and noncontrollable costs
D. direct and indirect costs
E. fixed and variable costs

Downloaded by Sky Park (parksky512@gmail.com)


lOMoARcPSD|32626796

7. When used for performance evaluation, periodic internal reports based on a responsibility
accounting system should not:
A. distinguish between controllable and noncontrollable costs
B. be related to the organization chart
C. include allocated fixed overhead
D. include variances between actual and budgeted controllable costs
E. all of the above

8. In the traditional view of responsibility accounting where individuals are evaluated rather than
operating systems, all of the following dysfunctional results may occur, except:
A. managers tend to take actions that are self-serving rather than beneficial to the company
as a whole
B. managers concentrate on meeting the budget rather than the best level of performance
that can be achieved
C. managers tend to focus their attention on long-run targets and ignore the short-term needs
of the company
D. many competent managers leave the company all of the above may occur

9. To avoid waste and maximize efficiency when transferring products among divisions in a
competitive economy, a large diversified corporation should base transfer prices on
A. variable cost.
B. market price.
C. full cost.
D. production cost.

10. The maximum of the transfer price negotiation range is


A. determined by the buying division.
B. set by the selling division.
C. influenced only by internal cost factors.
D. negotiated by the buying and selling division.

11. The presence of idle capacity in the selling division may increase
A. the incremental costs of production in the selling division.
B. the market price for the good.
C. the price that a buying division is willing to pay on an internal transfer.
D. a negotiated transfer price.

12. The minimum potential transfer price is determined by


A. incremental costs in the selling division.
B. the lowest outside price for the good.
C. the extent of idle capacity in the buying division.
D. negotiations between the buying and selling division.

13. As the internal transfer price is increased,


A. overall corporate profits increase.
B. profits in the buying division increase.
C. profits in the selling division increase.
D. profits in the selling division and the overall corporation increase.

14. To evaluate the performance of individual departments, interdepartmental transfers of a product


should preferably be made at prices
A. equal to the market price of the product.
B. set by the receiving department.
C. equal to fully-allocated costs of the producing department.
D. equal to variable costs to the producing department.

Downloaded by Sky Park (parksky512@gmail.com)


lOMoARcPSD|32626796

15. Allocating service department costs to revenue-producing departments is an alternative to


A. responsibility accounting.
B. the use of profit centers.
C. the use of cost centers.
D. a transfer pricing system.

Computer Solutions Corporation

Computer Solutions Corporation manufactures and sells various high-tech office automation products.
Two divisions of Office Products Inc. are the Computer Chip Division and the Computer Division.
The Computer Chip Division manufactures one product, a "super chip," that can be used by both the
Computer Division and other external customers. The following information is available on this
month's operations in the Computer Chip Division:

Selling price per chip $50


Variable costs per chip $20
Fixed production costs $60,000
Fixed SG&A costs $90,000
Monthly capacity 10,000 chips
External sales 6,000 chips
Internal sales 0 chips

Presently, the Computer Division purchases no chips from the Computer Chips Division, but instead
pays $45 to an external supplier for the 4,000 chips it needs each month.

16. Refer to Computer Solutions Corporation. Assume that next month's costs and levels of operations
in the Computer and Computer Chip Divisions are similar to this month. What is the minimum of
the transfer price range for a possible transfer of the super chip from one division to the other?
A. $50
B. $45
C. $20
D. $35

17. Refer to Computer Solutions Corporation. Assume that next month's costs and levels of operations
in the Computer and Computer Chip Divisions are similar to this month. What is the maximum of
the transfer price range for a possible transfer of the chip from one division to the other?
E. $50
F. $45
G. $35
H. $30

18. Refer to Computer Solutions Corporation. Two possible transfer prices (for 4,000 units) are under
consideration by the two divisions: $35 and $40. Corporate profits would be ___________ if $35 is
selected as the transfer price rather than $40.
A. $20,000 larger
B. $40,000 larger
C. $20,000 smaller
D. the same

Downloaded by Sky Park (parksky512@gmail.com)


lOMoARcPSD|32626796

19. Refer to Computer Solutions Corporation. If a transfer between the two divisions is arranged next
period at a price (on 4,000 units of super chips) of $40, total profits in the Computer Chip division
will
A. rise by $20,000 compared to the prior period.
B. drop by $40,000 compared to the prior period.
C. drop by $20,000 compared to the prior period.
D. rise by $80,000 compared to the prior period.

20. Refer to Computer Solutions Corporation. Assume, for this question only, that the Computer Chip
Division is selling all that it can produce to external buyers for $50 per unit. How would overall
corporate profits be affected if it sells 4,000 units to the Computer Division at $45? (Assume that
the Computer Division can purchase the super chip from an outside supplier for $45.)
A. no effect
B. $20,000 increase
C. $20,000 decrease
D. $90,000 increase

Diller Corporation
Diller Corporation has three production departments A, B, and C. Diller Corporation also has twoservice
departments, Administration and Personnel. Administration costs are allocated based on value
of assets employed, and Personnel costs are allocated based on number of employees. Assume that
Administration provides more service to the other departments than does the Personnel Department.

Dept. Direct Costs Employees Asset Value


Admin $900,000 25 $450,000
Personnel 350,000 10 600,000
A 700,000 15 300,000
B 200,000 5 150,000
C 250,000 10 800,000

21. Refer to Diller Corporation. Using the direct method, what amount of Administration costs is
allocated to A (round to the nearest dollar)?
A. $216,000
B. $150,000
C. $288,000
D. $54,000

22. Refer to Diller Corporation. Using the direct method, what amount of Personnel costs is allocated
to B (round to the nearest dollar)?
A. $50,000
B. $43,750
C. $26,923
D. $58,333

23. Refer to Diller Corporation. Using the direct method, what amount of Administration costs is
allocated to C (round to the nearest dollar)?
A. $576,000
B. $ 54,000
C. $108,000
D. $150,000

Downloaded by Sky Park (parksky512@gmail.com)


lOMoARcPSD|32626796

24. Refer to Diller Corporation. Using the step method, what amount of Administration costs is allocated
to Personnel (round to the nearest dollar)?
A. $72,973
B. $291,892
C. $145,946
D. $389,189

25. Refer to Diller Corporation. Using the step method, what amount of Administration costs is allocated
to A (round to the nearest dollar)?
A. $72,973
B. $291,892
C. $145,946
D. $389,189

26. Refer to Diller Corporation. Using the step method, what amount of Administration costs is allocated
to B (round to the nearest dollar)?
A. $72,973
B. $291,892
C. $145,946
D. $389,189

27. Refer to Diller Corporation. Using the step method, what amount of Administration costs is allocated
to C (round to the nearest dollar)?
A. $389,189
B. $145,946
C. $291,892
D. $72,973

28. Refer to Diller Corporation. Assume that Administration costs have been allocated and the balance
in Personnel is $860,000. What amount is allocated to A (round to the nearest dollar)?
A. $213,964
B. $106,982
C. $430,000
D. $0

29. Refer to Diller Corporation. Assume that Administration costs have been allocated and the balance
in Personnel is $860,000. What amount is allocated to B (round to the nearest dollar)?
A. $213,964
B. $430,000
C. $106,982
D. $143,333

30. Refer to Diller Corporation. Assume that Administration costs have been allocated and the balance
in Personnel is $860,000. What amount is allocated to C (round to the nearest dollar)?
A. $213,964
B. $430,000
C. $286,667
D. $143,333

Downloaded by Sky Park (parksky512@gmail.com)


lOMoARcPSD|32626796

1. In evaluating the performance of a profit center manager, the manager


A. and the sub-unit should be evaluated on the basis of the same costs and revenues.
B. should only be evaluated on the basis of variable costs and revenues of the sub-unit.
C. should be evaluated on all costs and revenues that are controllable by the manager
D. should be evaluated on all costs and revenues that can be directly traced to the sub-unit.

2. Return on investment (ROI) is a term most often used to express income earned on assets invested
in a business unit. A company's return on investment would increase if sales
A. increased by the same dollar amount as expenses and total assets increased.
B. remained the same and expenses were reduced by the same dollar amount that total assets
increased.
C. decreased by the same dollar amount that expenses increased.
D. and expenses increased by the same percentage that total assets increased.

3. Presently, the Classic Book Division of Griffin Publishing Corporation has a profit margin of 30%. If
total sales rise by $100,000, the net result will be
A. an increase in the profit margin ratio to above 30%.
B. a decrease in the profit margin ratio to below 30%.
C. no change in the profit margin ratio.
D. a change in the profit margin ratio that cannot be determined from this information.

4. Profit margin indicates the portion of sales that


A. covers fixed expenses.
B. is not used to cover expenses.
C. equals contribution margin.
D. equals product contribution margin.

5. If a new project generates a positive residual income, the


A. project's return on investment is less than the target rate.
B. project's return on investment is greater than the target rate.
C. project's return on investment is equal to the target rate.
D. relationship between the project's return on investment and the target rate cannot necessarily
be determined.

6. How would each of the following measures be affected if sales rise by $5,000 in the Boot Division?
ROI Asset turnover Profit margin
A. increase increase increase
B. increase no change increase
C. increase increase no change
D. no change no change increase

7. A prospective project under consideration by the Telephone Division of Communications


Corporation has an estimated residual income of $(20,000). If the project requires an investment
of $400,000, the
A. project generates a negative return on investment.
B. project's return on investment is zero.
C. project's return on investment is 5% less than the company's target rate.
D. company's target rate is 15%

Downloaded by Sky Park (parksky512@gmail.com)


lOMoARcPSD|32626796

8. Residual income is the


A. contribution margin of an investment center, less the imputed interest on the invested capital
used by the center.
B. contribution margin of an investment center, plus the imputed interest on the invested capital
used by the center.
C. income of an investment center, less the imputed interest on the invested capital used by the
center.
D. income of an investment center, plus the imputed interest on the invested capital used by the
center.

9. In selecting non-financial performance measures managers should choose measures that reflect
A. qualitative characteristics that point out sub-optimization activities and throughput bottlenecks.
B. both short-term and long-term measures related to critical success factors.
C. long-term supplier satisfaction levels.
D. short-term financial viability.

10. Relative to qualitative performance measures, quantitative performance measures are less
A. subject to manipulation.
B. dependent on accounting information.
C. effective in the pursuit of organizational goals.
D. subjective.

11. The Card Division of Party Company reported the following results for a recent year

Sales $8,000,000
Expenses 6,250,000
Total assets (1/1) 5,000,000
Total assets (12/31) 5,400,000

What was the profit margin for the Card Division?


A. 68%
B. 35%
C. 32%
D. 22%

12. The Card Division of Party Company reported the following results for a recent year

Sales $8,000,000
Expenses 6,250,000
Total assets (1/1) 5,000,000
Total assets (12/31) 5,400,000

What was the asset turnover ratio of the Card Division?


A. 1.538
B. 2.97
C. 0.650
D. 1.20

13. Empire Division of New York Delights, is evaluated based on residual income generated. In the
most recent year, the Empire Division generated a residual income of $2,000,000 and net income
of $5,000,000. The target rate of return for all divisions of New York Delights is 20%. What was the
return on investment for the Empire Division?
A. 40%
B. 13%
C. 20%
D. 33%

Downloaded by Sky Park (parksky512@gmail.com)


lOMoARcPSD|32626796

Texas Division of the Houston Company has the following statistics for its most recent operations:

Assets available for use (Market Value) $3,600,000


Assets available for use (Book Value) $2,000,000
Texas Division's return on investment 25%
Texas Division's residual income 200,000
Return on investment (entire Houston Company) 20%

14. Refer to Houston Company. Compute EVA assuming the cost of capital is 10% and the tax rate is
40%.
A. $ 90,000
B. $ 150,000
C. $0
D. $ (60,000)

15. Refer to Houston Company. What is the target rate of return in Houston Company?
A. 25%
B. 20%
C. 15%
D. 10%

16. Refer to Houston Company. If Houston Company evaluates its managers on the basis of return on
investment, the manager of Texas Division would invest in a project costing $100,000 only if it
increased net segment income by at least
A. $10,000.
B. $15,000.
C. $20,000.
D. $25,000.

17. Andersen Corporation has a target return of 15%. If a prospective investment has an estimated
return on investment of 20%, and a residual income of $10,000, what is the estimated cost of the
investment?
A. $200,000
B. $ 66,667
C. $ 50,000
D. The answer can't be determined from this information.

18. The Steelrod Division of Metal Products Company is considering an investment in a new project.
The project has an estimated cost of $1,000,000. If Metal Products Company has a target rate of
return of 12%, how large does the return on investment on this project need to be to generate
$150,000 of residual income?
A. 15%
B. 12%
C. 25%
D. 27%

19. In the South Division of Occident Company, segment income for the most recent year exceeded
residual income by $15,000. Also, return on investment exceeded the target rate of return by 10%.
What was the level of investment in the X Division for the most recent year?
A. $ 15,000
B. $100,000
C. $150,000
D. An answer can't be determined from this information.

Downloaded by Sky Park (parksky512@gmail.com)


lOMoARcPSD|32626796

20. The Steelrod Division of Metal Products Company is considering an investment in a new project.
The project has an estimated cost of $1,000,000. If Metal Products Company has a target rate of
return of 12%, how large does the return on investment on this project need to be to generate
$150,000 of residual income?
A. 15%
B. 12%
C. 25%
D. 27%

Downloaded by Sky Park (parksky512@gmail.com)


lOMoARcPSD|32626796

1. Capital budgeting tends to focus primarily on:


A. revenues.
B. costs.
C. cost centers.
D. programs and projects.
E. allocation tools.

2. Discounted-cash-flow analysis focuses primarily on:


A. the stability of cash flows.
B. the timing of cash flows.
C. the probability of cash flows.
D. the sensitivity of cash flows.
E. whether cash flows are increasing or decreasing.

3. In a net-present-value analysis, the discount rate is often called the:


A. payback rate.
B. hurdle rate.
C. minimal value.
D. net unit rate.
E. objective rate of return.

4. Consider the following factors related to an investment:


I. The net income from the investment.
II. The cash flows from the investment.
III. The timing of the cash flows from the investment.
Which of the preceding factors would be important considerations in a net-present-value
analysis?
A. I only.
B. II only.
C. I and II.
D. II and III.
E. I, II, and III.

5. The internal rate of return on an asset can be calculated:


A. if the return is greater than the hurdle rate.
B. if the asset's cash flows are identical to the future value of a series of cash flows.
C. if the future value of a series of cash flows can be arrived at by the annuity accumulation factor.
D. by finding a discount rate that yields a zero net present value.
E. by finding a discount rate that yields a positive net present value.

6. Depreciation is incorporated explicitly in the cash flow analysis of an investment proposal because
it:
A. is a cost of operations that cannot be avoided
B. results in an annual cash outflow
C. is a cash inflow
D. reduces the cash outlay for income taxes
E. represents the initial cash outflow spread over the life of the investment

Downloaded by Sky Park (parksky512@gmail.com)


lOMoARcPSD|32626796

7. Maxwell Company has an opportunity to acquire a new machine to replace one of its present
machines. The new machine would cost $90,000, have a 5-year life, and no estimated salvage
value. Variable operating costs would be $100,000 per year. The present machine has a book
value of $50,000 and a remaining life of 5 years. Its disposal value now is $5,000, but it would be
zero after 5 years. Variable operating costs would be $125,000 per year. Ignore income taxes.
Considering the 5 years in total, what would be the difference in profit before income taxes by
acquiring the new machine as opposed to retaining the present one?
A. $10,000 decrease
B. $15,000 decrease
C. $35,000 increase
D. $40,000 increase
E. none of the above

8. Assume that a project consists of an initial cash outlay of $100,000 followed by equal annual cash
inflows of $40,000 for 4 years. In the formula X = $100,000/$40,000, X represents the
A. payback period for the project.
B. profitability index of the project.
C. internal rate of return for the project.
D. project's discount rate.

9. Which of the following statements is true regarding capital budgeting methods?


A. The Fisher rate can never exceed a company's cost of capital.
B. The internal rate of return measure used for capital project evaluation has more
C. conservative assumptions than the net present value method, especially for projects that
generate a positive net present value.
D. The net present value method of project evaluation will always provide the same ranking of
projects as the profitability index method.
E. The net present value method assumes that all cash inflows can be reinvested at the project's
cost of capital.

10. Sensitivity analysis is


A. an appropriate response to uncertainty in cash flow projections.
B. useful in measuring the variance of the Fisher rate.
C. typically conducted in the post investment audit.
D. useful to compare projects requiring vastly different levels of initial investment.

11. Datasoft Industries is considering the purchase of a $100,000 machine that is expected to result in
a decrease of $15,000 per year in cash expenses. This machine, which has no residual value, has
an estimated useful life of 10 years and will be depreciated on a straight-line basis. For this
machine, the accounting rate of return would be
A. 10 percent.
B. 15 percent.
C. 30 percent.
D. 35 percent.

12. An investment project is expected to yield $10,000 in annual revenues, has $2,000 in fixed costs
per year, and requires an initial investment of $5,000. Given a cost of goods sold of 60 percent of
sales, what is the payback period in years?
A. 2.50
B. 5.00
C. 2.00
D. 1.25

Downloaded by Sky Park (parksky512@gmail.com)


lOMoARcPSD|32626796

13. A project has an initial cost of $100,000 and generates a present value of net cash inflows of
$120,000. What is the project's profitability index?
A. .20
B. 1.20
C. .80
D. 5.00

14. John Browning recently invested in a project that has an expected annual cash inflow of $7,000 for
10 years, and an expected payback period of 3.6 years. How much did John invest in the project?
A. $19,444
B. $36,000
C. $25,200
D. $40,000

15. The Wortham Corporation has recently evaluated a proposal to invest in cost-reducing production
technology. According to the evaluation, the project would require an initial investment of $17,166
and would provide equal annual cost savings for five years. Based on a 10 percent discount rate,
the project generates a net present value of $1,788. The project is not expected to have any salvage
value at the end of its five-year life.

Refer to Wortham Corporation. What are the expected annual cost savings of the project? Present
value tables or a financial calculator are required.
A. $3,500
B. $4,000
C. $4,500
D. $5,000

16. Webber Corporation is considering an investment in a labor-saving machine. Information on this


machine follows:

Cost $30,000
Salvage value in five years $0
Estimated life 5 years
Annual depreciation $6,000
Annual reduction in existing costs $8,000

Refer to Webber Corporation. What is the internal rate of return on this project (round to the nearest
1/2%)? Present value tables or a financial calculator are required.
A. 37.5%
B. 25.0%
C. 10.5%
D. 13.5%

Seabreeze Creations is considering an investment in a computer that is capable of producing various


images that are useful in the production of commercial art. The computer would cost $20,000 and have
an expected life of eight years. The computer is expected to generate additional annual net cash
receipts (before-tax) of $6,000 per year. The computer will be depreciated according to the straight-line
method and the firm's marginal tax rate is 25 percent.

17. Refer to Seabreeze Creations. What is the after-tax payback period for the computer project?
A. 7.62 years
B. 3.90 years
C. 4.44 years
D. 3.11 years

Downloaded by Sky Park (parksky512@gmail.com)


lOMoARcPSD|32626796

18. Refer to Seabreeze Creations. What is the after-tax net present value of the proposed project (using
a 16 percent discount rate)? Present value tables or a financial calculator are required.
A. $2,261
B. $(454)
C. $6,062
D. $(4,797)

Fleming Company is considering an investment in a machine that would reduce annual labor costs by
$30,000. The machine has an expected life of 10 years with no salvage value. The machine would be
depreciated according to the straight-line method over its useful life. The company's marginal tax rate
is 30 percent.

19. Refer to Fleming Company. Assume that the company will invest in the machine if it generates an
internal rate of return of 16 percent. What is the maximum amount the company can pay for the
machine and still meet the internal rate of return criterion? Present value tables or a financial
calculator are required.
A. $180,000
B. $210,000
C. $187,500
D. $144,996

20. Refer to Fleming Company. Assume the company pays $250,000 for the machine. What is the
expected internal rate of return on the machine? Present value tables or a financial calculator are
required.
A. between 8 and 9 percent
B. between 3 and 4 percent
C. between 17 and 18 percent
D. less than 1 percent

Downloaded by Sky Park (parksky512@gmail.com)

You might also like